Download as pdf or txt
Download as pdf or txt
You are on page 1of 68

Surgical Critical Care And Emergency

Surgery: Clinical Questions And


Answers 3rd Edition Forrest "Dell"
Moore (Editor)
Visit to download the full and correct content document:
https://ebookmass.com/product/surgical-critical-care-and-emergency-surgery-clinical-
questions-and-answers-3rd-edition-forrest-dell-moore-editor/
25.1 mm 216 x 276 mm

Rodriguez
Third Edition

Moore
Rhee
Surgical Critical Care and
Emergency Surgery Surgical

and
Critical Care

Surgical Critical Care


Emergency Surgery
In this newly revised edition of Surgical Critical Care and Emergency Surgery, a distinguished
team of emergency surgeons deliver a one-of-a-kind question-and-answer book on the growing

and Emergency
specialty of surgical critical care and acute surgery, ideal for those caring for the critically ill and
injured surgical patient.

This book reviews surgical critical care, emergency surgery, burns and trauma, and includes full

Surgery
color, high-quality surgical photographs to aid understanding. Readers will also benefit from access
to a website that offers additional topics and tests, as well as an archive of all test questions and
answers from previous editions.

The authors’ focus throughout is on the unique problems and complexity of illnesses of the

Clinical Questions
critically ill and injured surgical patient, and the specialist daily care that such patients require.

Perfect for the acute care surgeon, surgical intensivist and those in training, Surgical Critical Care
and Emergency Surgery will also earn a place in the libraries of those working in or with an interest
in critical care. and Answers
Forrest “Dell” Moore, MD, is Associate Professor of Surgery at the TCU & UNTHSC School
of Medicine, and is Vice Chief of Surgery and Associate Trauma Medical Director at John Peter
Smith Hospital in Fort Worth, Texas, USA.

Peter M. Rhee, MD, is Professor of Surgery, New York Medical College, and Chief of Acute Care

Third Edition
Surgery, Westchester Medical Center, Valhalla, NY, USA.

Carlos J. Rodriguez, DO, is Associate Professor of Surgery at the TCU & UNTHSC School of
Medicine, and is Director of Emergency General Surgery and Surgical Research at John Peter Smith Edited by
Hospital in Fort Worth, Texas, USA.
Forrest “Dell” Moore
Peter M. Rhee
A companion website with additional resources is available at
www.wiley.com/go/surgicalcriticalcare3e Carlos J. Rodriguez
Cover Design: Wiley
Cover Image: Courtesy of Peter Rhee

www.wiley.com
Surgical Critical Care and Emergency Surgery
Surgical Critical Care and Emergency Surgery

Clinical Questions and Answers

Third Edition

Edited by

Forrest “Dell” Moore, MD, FACS


Associate Professor of Surgery
TCU & UNTHSC School of Medicine
Vice Chief of Surgery
Associate Trauma Medical Director
John Peter Smith Health
Forth Worth, TX, USA

Peter M. Rhee, MD, MPH, FACS, FCCM, DMCC


Professor of Surgery at New York Medical College
USUHS, and Morehouse College of Medicine
Chief of Acute Care Surgery and Trauma
Vice Chair of Surgery

Carlos J. Rodriguez, DO
Associate Professor of Surgery
TCU & UNTHSC School of Medicine
Director, Emergency General Surgery
Director, Surgical Research
John Peter Smith Health
Fort Worth, TX, USA
This third edition first published 2022
© 2022 John Wiley & Sons Ltd

Edition History
John Wiley & Sons Ltd (1e, 2012; 2e, 2018)

All rights reserved. No part of this publication may be reproduced, stored in a retrieval system, or transmitted, in any form or by any means,
electronic, mechanical, photocopying, recording or otherwise, except as permitted by law. Advice on how to obtain permission to reuse material
from this title is available at http://www.wiley.com/go/permissions.

The right of Forrest “Dell” Moore, Peter M. Rhee, and Carlos J. Rodriguez to be identified as the authors of the editorial material in this work has
been asserted in accordance with law.

Registered Offices
John Wiley & Sons, Inc., 111 River Street, Hoboken, NJ 07030, USA
John Wiley & Sons Ltd, The Atrium, Southern Gate, Chichester, West Sussex, PO19 8SQ, UK

Editorial Office
9600 Garsington Road, Oxford, OX4 2DQ, UK

For details of our global editorial offices, customer services, and more information about Wiley products visit us at www.wiley.com.

Wiley also publishes its books in a variety of electronic formats and by print-­on-­demand. Some content that appears in standard print versions of
this book may not be available in other formats.

Limit of Liability/Disclaimer of Warranty


The contents of this work are intended to further general scientific research, understanding, and discussion only and are not intended and should
not be relied upon as recommending or promoting scientific method, diagnosis, or treatment by physicians for any particular patient. In view
of ongoing research, equipment modifications, changes in governmental regulations, and the constant flow of information relating to the use of
medicines, equipment, and devices, the reader is urged to review and evaluate the information provided in the package insert or instructions for
each medicine, equipment, or device for, among other things, any changes in the instructions or indication of usage and for added warnings and
precautions. While the publisher and authors have used their best efforts in preparing this work, they make no representations or warranties with
respect to the accuracy or completeness of the contents of this work and specifically disclaim all warranties, including without limitation any
implied warranties of merchantability or fitness for a particular purpose. No warranty may be created or extended by sales representatives, written
sales materials or promotional statements for this work. The fact that an organization, website, or product is referred to in this work as a citation
and/or potential source of further information does not mean that the publisher and authors endorse the information or services the organization,
website, or product may provide or recommendations it may make. This work is sold with the understanding that the publisher is not engaged
in rendering professional services. The advice and strategies contained herein may not be suitable for your situation. You should consult with a
specialist where appropriate. Further, readers should be aware that websites listed in this work may have changed or disappeared between when
this work was written and when it is read. Neither the publisher nor authors shall be liable for any loss of profit or any other commercial damages,
including but not limited to special, incidental, consequential, or other damages.

Library of Congress Cataloging-­in-­Publication Data

Names: Moore, Forrest “Dell”. editor. | Rhee, Peter M., 1961- editor. | Rodriguez,
Carlos J., editor.
Title: Surgical critical care and emergency surgery : clinical questions
and answers / edited by Forrest “Dell”. Moore, Peter M. Rhee, Carlos J.
Rodriguez.
Description: Third edition. | Hoboken, NJ : Wiley-Blackwell, 2022. |
Includes bliographical references and index.
Identifiers: LCCN 2021029654 (print) | LCCN 2021029655 (ebook) | ISBN
9781119756750 (paperback) | ISBN 9781119756767 (adobe pdf ) | ISBN
9781119756774 (epub)
Subjects: MESH: Critical Care–methods
https://id.nlm.nih.gov/mesh/D003422Q000379 | Surgical Procedures,
Operative–methods | Critical Illness–therapy | Emergencies | Emergency
Treatment–methods | Wounds and Injuries–surgery | Examination
Questions
Classification: LCC RD93 (print) | LCC RD93 (ebook) | NLM WO 18.2 | DDC
617/.026–dc23
LC record available at https://lccn.loc.gov/2021029654
LC ebook record available at https://lccn.loc.gov/2021029655

Cover Design: Wiley


Cover Image: Courtesy of Peter Rhee

Set in 10/12pt Warnock Pro by Straive, Pondicherry, India

10 9 8 7 6 5 4 3 2 1
Contents

List of Contributors ix
About the Companion Website xii

Part One Surgical Critical Care 1

1 Respiratory and Cardiovascular Physiology 3


Anne Warner, MD, Harsh Desai, MD, and Frederick Giberson, MD

2 Cardiopulmonary Resuscitation, Oxygen Delivery and Shock 11


Kevin W. Cahill, MD, Harsh Desai, MD, and Luis Cardenas, DO, PhD

3 ECMO 19
Mauer Biscotti III, MD, Matthew A. Goldshore, MD, PhD, MPH, and Jeremy W. Cannon, MD, SM

4 Arrhythmias, Acute Coronary Syndromes, and Hypertensive Emergencies 33


Ryan Malcom, MD

5 Sepsis and the Inflammatory Response to Injury 45


Ilya Shnaydman, MD and Matthew Bronstein, MD

6 Hemodynamic and Respiratory Monitoring 55


Jared Sheppard, MD, Christopher S. Nelson, MD, and Stephen L. Barnes, MD

7 Airway and Perioperative Management 67


Jared Sheppard, MD, Jeffrey P. Coughenour, MD, and Stephen L. Barnes, MD

8 Acute Respiratory Failure and Mechanical Ventilation 77


Adrian A. Maung, MD and Lewis J. Kaplan, MD

9 Infectious Disease 85
Rathnayaka M. K. Gunasingha, MD, Patrick Benoit, DO, and Matthew J. Bradley, MD

10 Pharmacology and Antibiotics 99


Michelle Strong, MD, PhD and Elaine Cleveland, MD

11 Transfusion, Hemostasis, and Coagulation 113


Lindsey Karavites, MD and Kazuhide Matsushima, MD

12 Analgesia and Anesthesia 123


Toni Manougian, MD, MBA and Bardiya Zangbar, MD

13 Delirium, Alcohol Withdrawal and Psychiatric Disorders 137


Thomas Muse, MD and Rondi Gelbard, MD
vi Contents

14 Acid-­Base, Fluids, and Electrolytes 145


Joshua Dilday, DO, Catherine Cameron, MD, and Christopher Bell, MD

15 Metabolic Illness and Endocrinopathies 155


Andrew J. Young, MD and Therese M. Duane, MD

16 Hypothermia and Hyperthermia 165


Drew Farmer, MD and Shariq Raza, MD

17 Acute Kidney Injury 173


Cassandra Q. White, MD and Terence O’Keeffe, MB ChB

18 Liver Failure 181


Bellal Joseph, MD and Omar Obaid, MD

19 Nutrition Support in Critically Ill Patients 189


Ida Molavi, MD and Jorge Con, MD

20 Neurocritical Care 203


Kevin W. Cahill, MD and Frederick Giberson, MD

21 Venous Thromboembolism 213


Brett M. Chapman, MD and Herb A. Phelan, MD, MSCS

22 Transplantation, Immunology, and Cell Biology 225


Jarrett Santorelli, MD and Leslie Kobayashi, MD

23 Obstetric Critical Care 237


Gary Lombardo, MD

24 Pediatric Critical Care 251


Juan P. Gurria, MD and J. Craig Egan, MD

25 Envenomation, Poisoning, and Toxicology 261


Michelle Strong, MD, PhD and Elaine Cleveland, MD

26 Common Procedures in the ICU 273


Fariha Sheikh, MD and Adam D. Fox, DO, DPM

27 Diagnostic Imaging, Ultrasound and Interventional Radiology 281


Hang Ho, MD and Terence O’Keeffe, MB ChB

Part Two Emergency Surgery 293

28 Neurotrauma 295
Bellal Joseph, MD and Raul Reina Limon, MD

29 Blunt and Penetrating Neck Trauma 307


Eric Raschke, DO and Leslie Kobayashi, MD

30 Cardiothoracic and Thoracic Vascular Injury 319


Charles J. Fox, MD and Annalise Penikis, MD

31 Abdominal and Abdominal Vascular Injury 333


Melike Harfouche, MD and Joseph DuBose, MD
Contents vii

32 Orthopedic and Hand Trauma 349


Brett D. Crist, MD and Gregory J. Della Rocca, MD, PhD

33 Peripheral Vascular Trauma 359


Yousef Abuhakmeh, DO and Jonathan Swisher, MD

34 Urologic Trauma and Disorders 369


Daniel Roubik, MD and Luke Hofmann, DO

35 Care of the Pregnant Trauma Patient 379


Navdeep Samra, MD and Jaideep Sandhu, MBBS, MPH

36 Esophagus, Stomach, and Duodenum 391


Collin Stewart, MD and Andrew Tang, MD

37 Small Intestine, Appendix, and Colorectal 403


Elise Sienicki, MD, Vishal Bansal, MD, and Jay J. Doucet, MD, MSc

38 Gallbladder and Pancreas 415


Kirstie Jarrett, MD and Andrew Tang, MD

39 Liver and Spleen 429


Narong Kulvatunyou, MD and Peter M. Rhee, MD

40 Incarcerated Hernias and Abdominal Wall Reconstruction 443


Michael C. Smith, MD and Richard S. Miller, MD

41 Necrotizing Soft Tissue Infections and Other Soft Tissue Infections 451
MAJ Jacob Swann, MD and Joseph DuBose, MD

42 Obesity and Bariatric Surgery 459


Thomas A. O’Hara, DO and Gregory S. Peirce, MD

43 Burns, Inhalational Injury, and Lightning Injury 471


MAJ Jacob Swann, MD and William Mohr, III, MD

44 Gynecologic Surgery 481


Joshua Klein, DO

45 Cardiovascular and Thoracic Surgery 493


Kristine Tolentino Parra, MD, Theodore Pratt, MD, and Matthew J. Martin, MD

46 Pediatric Surgery 505


Brandt Sisson, MD, Matthew J. Martin, MD, and Romeo Ignacio, MD

47 Geriatrics 523
Douglas James, MD and Kartik Prabhakaran, MD

48 Statistics 537
Alan Cook, MD, MS

49 Ethics, End-­of-­Life, and Organ Retrieval 549


Lewis J. Kaplan, MD

Index 559
ix

List of Contributors

Yousef Abuhakmeh, DO Catherine Cameron, MD


MAJ, MC US Army Landstuhl Regional Medical Center
Banner University Medical Center Landstuhl, Germany
University of Arizona College of Medicine
Tucson, AZ, USA Jeremy W. Cannon, MD, SM
Division of Traumatology, Surgical Critical Care &
Vishal Bansal, MD Emergency Surgery, Perelman School of Medicine at the
Scripps Mercy Hospital University of Pennsylvania
San Diego, CA, USA Philadelphia, PA, USA
Stephen L. Barnes, MD Department of Surgery, F. Edward Hébert School
Division of Acute Care Surgery, Department of Surgery of Medicine, Uniformed Services University of the
University of Missouri Health Sciences
Columbia, MO, USA Bethesda, MD, USA

Elise Becker, MD Luis Cardenas, DO, PhD


Naval Medical Center Department of Surgery
San Diego, CA, USA Christiana Care Health Care System
Newark, DE, USA
Christopher Bell, MD
William Beaumont Army Medical Center Brett M. Chapman, MD
El Paso, TX, USA LSUHSC-­New Orleans
New Orleans, LA, USA
Patrick Benoit, DO
Walter Reed National Military Medical Center Elaine Cleveland, MD
Bethesda, MD, USA William Beaumont Army Medical Center
Mauer Biscotti III, MD El Paso, TX, USA
Division of General Surgery, Department of Surgery
Jorge Con, MD
San Antonio Military Medical Center
Division of Trauma and Acute Care Surgery
San Antonio, TX, USA
New York Medical College
Matthew J. Bradley, MD Westchester Medical Center
Uniformed Services University of the Health Sciences Valhalla, NY, USA
Program Director General Surgery Residency
Walter Reed National Military Medical Center Alan Cook, MD, MS
Bethesda, MD, USA University of Texas at Tyler
Tyler, TX, USA
Matthew Bronstein, MD
Division of Trauma and Acute Care Surgery J. Craig Egan, MD
New York Medical College Phoenix Children’s Hospital
Westchester Medical Center Phoenix, AZ, USA
Valhalla, NY, USA
Brett D. Crist, MD
Kevin W. Cahill, MD Department of Orthopaedic Surgery
Christiana Care Health Care System University of Missouri
Newark, DE, USA Columbia, MO, USA
x List of Contributors

Jeffrey P. Coughenour, MD Rondi Gelbard, MD


Division of Acute Care Surgery Division of Trauma and Acute Care Surgery
Department of Surgery Department of Surgery
University of Missouri University of Alabama at Birmingham
Columbia, MO, USA Birmingham, AL, USA

Gregory J. Della Rocca, MD, PhD Frederick Giberson, MD


Department of Orthopaedic Surgery Department of Surgery
University of Missouri Christiana Care Health Care System
Columbia, MO, USA Newark
DE, USA
Harsh K. Desai, MD
Matthew A. Goldshore, MD, PhD, MPH
Department of Surgery
Department of Surgery
Christiana Care Health Care System
Perelman School of Medicine at the University
Newark, DE, USA
of Pennsylvania
Joshua Dilday, DO Philadelphia, PA, USA
Division of Acute Care Surgery Rathnayaka M. K. Gunasingha, MD
University of Southern California Walter Reed National Military Medical Center
LAC + USC Medical Center Bethesda, MD, USA
Los Angeles, CA, USA
Juan P. Gurria, MD
Jay J. Doucet, MD, MSc Phoenix Children’s Hospital
Department of Surgery Phoenix, AZ, USA
University of California San Diego
Melike Harfouche MD
San Diego, CA, USA
Division of Trauma and Acute Care Surgery
Therese M. Duane, MD University of Maryland – Shock Trauma Center
TCU & UNTHSC School of Medicine Baltimore, MD, USA
Department of Surgery Hang Ho, MD
Texas Health Resources Augusta University Medical Center
Fort Worth, TX, USA Augusta, GA, USA
Joseph DuBose, MD
Luke Hofmann, DO
Department of Surgery Brooke Army Medical Center
Dell School of Medicine San Antonio, TX, USA
University of Texas Austin F. Edward Hebert School of Medicine Uniformed
Austin, TX, USA
Services University
Drew Farmer, MD Bethesda, MD, USA
Trauma Surgery, Surgical Critical Care Romeo Ignacio, MD
& Emergency Surgery Division of Pediatric Surgery
Perelman School of Medicine Rady Children’s Hospital
University of Pennsylvania San Diego, CA, USA
Philadelphia, PA, USA
MAJ Jacob Swann, MD
Adam D. Fox, DO
Regions Hospital
Division of Trauma and Critical Care Surgery Saint Paul, MN, USA
Rutgers New Jersey Medical School
University Hospital Douglas James, MD
Newark, NJ, USA Section of Trauma and Acute Care Surgery
Westchester Medical Center
Charles J. Fox, MD Valhalla, NY, USA
R Adams Cowley Shock Trauma Center
Division of Vascular Surgery Kirstie Jarrett, MD
University of Maryland School of Medicine Banner University Medical Center
Baltimore, MD, USA Tucson, AZ, USA
List of Contributors xi

Bellal Joseph, MD Toni Manougian MD, MBA


Division of Trauma, Surgical Critical Care, Burns and Department of Critical Care Anesthesiology
Acute Care Surgery New York Medical College
University of Arizona College of Medicine Westchester Medical Center
Banner University Medical Center Valhalla, NY, USA
Tucson, AZ, USA
Matthew J. Martin, MD
Lewis J. Kaplan, MD, FCCM, FCCP Trauma and Acute Care Surgery Service
Division of Trauma, Surgical Critical Care and Scripps Mercy Hospital
Emergency Surgery, Department of Surgery, Perelman San Diego, CA, USA
School of Medicine, University of Pennsylvania
Philadelphia, PA, USA Kazuhide Matsushima, MD
Division of Acute Care Surgery
Corporal Michael J. Crescenz VA Medical Center University of Southern California
Philadelphia, PA, USA LAC+USC Medical Center
Lindsey Karavites, MD Los Angeles, CA, USA
Division of Acute Care Surgery Adrian A. Maung, MD , FCCM
University of Southern California Yale School of Medicine
LAC+USC Medical Center New Haven, CT, USA
Los Angeles, CA, USA
Richard S. Miller, MD
Joshua Klein, DO Department of Surgery
Department of Surgery, Trauma & Acute Care Surgeon TCU & UNTHSC School of Medicine
Westchester Medical Center, Division of Trauma John Peter Smith Health
& Acute Care Surgery Fort Worth, TX, USA
New York Medical College, Valhalla, NY, USA
William Mohr III, MD
Leslie Kobayashi, MD Regions Hospital
Division of Trauma, Acute Care Surgery Saint Paul, MN, USA
Surgical Critical Care and Burns
University of California San Diego Ida Molavi, MD
San Diego, CA, USA Department of Trauma and Acute Care Surgery
Louisiana State University Health
Narong Kulvatunyou, MD Shreveport, LA, USA
Department of Surgery
University of Arizona School of Medicine Thomas Muse, MD
Banner University Medical Center Department of Trauma and Acute Care Surgery
Tucson, AZ, USA Department of Surgery
University of Alabama at Birmingham
Raul Reina Limon, MD Birmingham, AL, USA
Division of Trauma, Critical Care, Burns, and
Emergency Surgery Christopher S. Nelson, MD
Department of Surgery Division of Acute Care Surgery
University of Arizona Department of Surgery
Tucson, AZ, USA University of Missouri
Columbia, MO, USA
Gary Lombardo, MD
Division of Trauma and Acute Care Surgery Omar Obaid, MD
New York Medical College Division of Trauma, Critical Care
Westchester Medical Center Burns, and Emergency Surgery
Valhalla, NY, USA Department of Surgery
University of Arizona
Ryan Malcom, MD Tucson, AZ, USA
Division of Trauma and Acute Care Surgery
New York Medical College Thomas A. O’Hara, DO
Westchester Medical Center Dwight D. Eisenhower Army Medical Center
Valhalla, NY, USA Fort Gordon, GA, USA
xii List of Contributors

Terence O’Keeffe, MB ChB Jaideep Sandhu, MBBS, MPH


Augusta University Medical Center City of Hope National Medical Center
Augusta, GA, USA Duarte, CA, USA

Kristine Tolentino Parra, MD Jarrett Santorelli, MD


Naval Medical Center Division of Trauma, Acute Care Surgery, Surgical
San Diego, CA, USA Critical Care and Burns
University of California San Diego
Gregory S. Peirce, MD San Diego, CA, USA
Womack Army Medical Center
Fort Bragg, NC, USA Fariha Sheikh, MD
Division of Trauma and Critical Care Surgery
Annalise Penikis, MD Rutgers New Jersey Medical School
University of Maryland Medical Center University Hospital
Baltimore, MD, USA Newark, NJ, USA

Herb A. Phelan, MD, MSCS Jared Sheppard, MD


Department of Surgery, LSU School of Medicine Division of Acute Care Surgery, Department of Surgery
New Orleans, LA, USA University of Missouri
Columbia, MO, USA
Kartik Prabhakaran, MD
New York Medical College Ilya Shnaydman, MD
Westchester Medical Center Division of Trauma and Acute Care Surgery
Valhalla, NY, USA New York Medical College
Westchester Medical Center
Theodore Pratt, MD Valhalla, NY, USA
Naval Medical Center
San Diego, CA, USA Elise Sienicki, MD
Naval Medical Center, San Diego, CA, USA
Eric Raschke, DO
Madigan Army Medical Center Brandt Sisson, MD
Tacoma, WA, USA Naval Medical Center
Shariq Raza, MD San Diego, CA, USA
Trauma Surgery, Surgical Critical Care Michael C. Smith, MD
& Emergency Surgery Division of Trauma and Surgical Critical Care
Perelman School of Medicine Vanderbilt University Medical Center
University of Pennsylvania Nashville, TN, USA
Philadelphia, PA, USA
Collin Stewart, MD
Peter M. Rhee, MD
Banner University Medical Center
Division of Trauma and Acute Care Surgery
University of Arizona College of Medicine
New York Medical College
Tucson, AZ, USA
Westchester Medical Center
Valhalla, NY, USA
Michelle Strong, MD, PhD
Daniel Roubik, MD Trauma and Acute Care Surgeon
Brooke Army Medical Center Austin, TX, USA
San Antonio, TX, USA
Jonathan Swisher, MD
Navdeep Samra, MD LTC, MC US Army
LSU Health William Beaumont Army Medical Center
Shreveport, LA, USA El Paso, TX, USA
List of Contributors xiii

Andrew Tang, MD Andrew J. Young, MD


University of Arizona College of Medicine Division of Trauma, Critical Care and Burn
Banner University Medical Center The Ohio State University
Tucson, AZ, USA Columbus, OH, USA
Anne Warner, MD Bardiya Zangbar, MD
Department of Surgery Division of Trauma and Acute Care Surgery
Christiana Care Health Care System New York Medical College
Newark, DE, USA Westchester Medical Center
Valhalla, NY, USA
Cassandra Q. White, MD
Department of Surgery
Augusta University
Augusta, GA, USA
xiv

About the Companion Website

This book is accompanied by a companion website

www.wiley.com/go/surgicalcriticalcare3e

The website features:


●● Interactive multiple choice questions
1

Part One

Surgical Critical Care


3

Respiratory and Cardiovascular Physiology


Anne Warner, MD, Harsh Desai, MD, and Frederick Giberson, MD
Department of Surgery, Christiana Care Health Care System, Newark, DE, USA

1 I n a patient who develops ARDS, the addition of PEEP 2 W


 hich of the following is NOT a component of the
in optimizing ventilatory support has which of the fol- inflammatory cascade leading to lung injury in
lowing effects? ARDS?
A Maximal alveolar recruitment with inspiration. A Injury to type I and type II epithelial cells within
B Decreasing mean airway pressure. the alveoli.
C Decreased right ventricular afterload. B Capillary endothelial dysregulation resulting in
D Improvement of functional residual capacity (FRC). recruitment of neutrophils.
E Increasing left ventricular afterload. C Sequestration of predominantly lymphocytes
within the pulmonary microcirculation.
The use of positive end-­expiratory pressure (PEEP) as D Release of cytoplasmic granules from neutrophil
part of the ARDS ventilatory strategy has been shown to degranulation.
improve the functional residual capacity (FRC) above the E Exudation of protein-­ rich fluid into the distal
closing pressure of alveoli, thereby preventing alveolar airspaces.
collapse. PEEP maximizes alveolar recruitment at end
expiration, not inspiration. The addition of PEEP The inflammatory cascade in ARDS is thought to be ini-
increases inflation pressure, thereby increasing peak tiated by activation of circulating neutrophils by the
alveolar pressure and ultimately mean airway pressure. release of IL-­1 and TNF by macrophages and monocytes.
Increased PEEP increases pulmonary vascular resistance Endothelial dysregulation attracts and retains neutro-
impeding right vascular stroke volume and thereby left phils with subsequent sequestration within the pulmo-
ventricular filling. It also decreases the transmural pres- nary microcirculation. This occurs through adhesion of
sure – the pressure needed to be overcome in order to neutrophils to endothelial cells and neutrophil stiffening.
eject stroke volume – thereby decreasing left ventricular Neutrophils then move into lung parenchyma and
afterload. degranulate propagating injury to the type I and II epi-
thelial cells within the alveoli allowing for exudation of
Answer: D
protein-­rich fluid, erythrocytes, and platelets into the
Briel M, Meade M, Mercat A, et al. Higher vs lower distal airspaces.
positive end-­expiratory pressure in patients with acute Answer: C
lung injury and acute respiratory distress syndrome.
JAMA. 2010; 303 (9): 865–873. Abraham E. Neutrophils and acute lung injury. Crit Care
Schmitt JM, Viellard-­Baron A, Augarde R, et al. Positive Med. 2003; 31(supp): S195–S199.
end-­expiratory pressure titration in acute respiratory
distress syndrome patients: impact on right ventricular 3 A
 27-­year-­old man is undergoing exploratory lapa-
outflow impedance evaluated by pulmonary artery rotomy after presenting with a gunshot wound to the
Doppler flow velocity measurements. Crit Care Med. left flank. He is currently hemodynamically stable.
2001; 29: 1154–1158. The operative team has concern for possible ureteral

Surgical Critical Care and Emergency Surgery: Clinical Questions and Answers, Third Edition.
Edited by Forrest “Dell” Moore, Peter M. Rhee, and Carlos J. Rodriguez.
© 2022 John Wiley & Sons Ltd. Published 2022 by John Wiley & Sons Ltd.
Companion website: www.wiley.com/go/surgicalcriticalcare3e
4 Surgical Critical Care and Emergency Surgery

injury and asks that methylene blue be administered history of COPD. Which of the following pulmonary
for identification of possible urine leak. Shortly after function test patterns would be expected in a patient
administration, the patient desaturates to SpO2 of with COPD?
82% with remaining hemodynamics remaining appro- A FEV1 decreased; FVC decreased/normal; FEV1/
priate. What is the management for the etiology of this FVC ratio decreased.
patient’s desaturation event? B FEV1 increased; FVC decreased; FEV1/FVC ratio
A Perform a left tube thoracostomy. increased.
B Immediate bronchoscopy. C FEV1 decreased/normal; FVC decreased; FEV1/
C Abort the procedure. FVC ratio normal.
D Manual bag mask ventilation. D FEV1 increased; FVC increased; FEV1/FVC ratio
E Watch and wait without immediate intervention. increased.
E FEV1 decreased; FVC decreased; FEV1/FVC ratio
The multiple uses of methylene blue have been estab- decreased.
lished including use in methemoglobinemia treatment as
well as potential use in vasoplegic syndrome. In the oper- Pulmonary function testing is often used in preoperative
ating room, methylene blue is often used to evaluate evaluation, particularly prior to thoracic procedures.
renal function and for potential leak in urologic proce- These can be used, in addition to history and exam, to
dures. However, one of the adverse effects of methylene identify obstructive versus restrictive lung processes.
blue is to decrease pulse oximetry readings. Three of the important measures are the forced vital
Pulse oximeters are made up of a side containing two capacity (FVC) – the total volume forcefully expired
light emitting diodes that emit at 660nm and 940nm after maximal inspiratory effort; forced expiratory vol-
detecting deoxygenated and oxygenated hemoglobin, ume in 1 second (FEV1) – the volume of air forcefully
respectively. The light is captured after passing through expired after maximal inspiratory effort in 1 second; the
the arteries in the finger by a probe on the other side of FEV1/FVC ratio. In evaluating spirometry results, first
the oximeter. This is then passed through and alternating step is to interpret the FEV1/FVC ratio. If less than the
current amplifier to block nonpulsatile wave forms from lower limit of normal, an obstructive pattern is sus-
veins. The ratio of oxygenated to total hemoglobin is pected. If greater than lower limit of normal, the FVC is
used to calculate SpO2. When administered, methylene evaluated and if less than lower limit of normal, a restric-
blue transiently decreases the detected oxygenated tive process is considered. Obstructive diseases include
hemoglobin as the methemoglobin fraction, usually a COPD, asthma, and emphysema while restrictive lung
small percentage of total circulating hemoglobin, diseases include neuromuscular disorders and intersti-
increases until processed out through the renal system. tial lung diseases.
Therefore, for this patient, aborting the procedure is not
Answer: A
necessary. The desaturation is transient and not caused
by mucus plugging, which may require bronchoscopy, Barreiro TJ and Perillo I. An approach to interpreting
pneumothorax, which would require tube thoracostomy, spirometry. Am Fam Physician. 2004; 69(5): 1107–1115.
or significant atelectasis, which may require bag mask Pellegrino R, Viegi G, Brurasco V, et al. Interpretative
ventilation. strategies for lung function tests. Eur Respir J. 2005;
Answer: E 26:948–968.

Clifton J and Leikin JB. Methylene blue. Am J Ther. 2003; 5 Y


 ou are caring for a patient in your SICU who is post
10(4): 289–291. total abdominal colectomy and end ileostomy.
Rong LQ, Mauer E, Mustapich TL, et al. Characterization Ileostomy output has been in excess of 1.5L daily
of the rapid drop in pulse oximetry reading after with concomitant acute kidney injury noted on basic
intraoperative administration of methylene blue in open metabolic panel with continued required resuscita-
thoracoabdominal aortic repairs. Anesth Analg. 2019; tion. Which of the following represents the primary
129(5): 142–145. relationships between alveolar pressure (PA), pulmo-
nary arterial pressure (Pa), and pulmonary venous
4 A
 65-­year-­old woman is in the post-­anesthesia care pressure (P v) within the lung in a state of
unit following elective inguinal hernia surgery. Shortly hypovolemia?
after arriving, she is noted to have increasing short- A Pa > Pv > PA and PA > Pv > Pa
ness of breath and wheezing requiring administration B PA > Pv > Pa and Pa > PA > Pv
of a nebulized beta agonist. The patient has a known C Pa > PA > Pv and PA > Pa > Pv
Respiratory and Cardiovascular Physiology 5

D PA > Pa > Pv and Pv > Pa > PA extrinsic compression (Pa > Pv > PA). In a hypovolemic
E Pa > PA > Pv and Pa > Pv > PA individual, as in this patient, decreased circulating vol-
ume converts Zone 3 tissue to Zone 1 and 2, increasing
The relationship between alveolar pressure, pulmonary dead space.
arterial pressure, and pulmonary venous pressure repre- Answer: C
sents the West zones of the lung. Zone 1, not seen in nor-
mal physiology, signifies alveolar dead space secondary West JB and Dollery CT. Distribution of blood flow and
to increased alveolar pressure causing arterial collapse the pressure-­flow relations of the whole lung. J Appl
(PA > Pa > Pv). Zone 2 represents pulsatile perfusion Physiol. 1965; 20(2): 175–183.
(Pa > PA > Pv) typically the upper portions of lung in a
typical, upright person. Zone 3 represents the bulk of For questions 7–10, use the following figure to match the
healthy lung tissue with continuous blood flow without clinical scenario to the appropriate flow volume loop:

A B C D E

Expiration
Flow (L/second)

Inspiration
TLC RV TLC RV TLC RV TLC RV TLC RV

Volume

6 A
 42-­year-­old man presents to the ICU following intu- Intrathoracic variable o ­ bstruction, such as with bron-
bation for COPD exacerbation. chogenic cysts or intrathoracic tracheomalacia, is
demonstrated by flattening of the expiratory compo-
7 A
 n 18-­year-­old woman diagnosed on bronchoscopy nent, as seen in loop C. Pleural pressure becomes posi-
with intratracheal lipoma. tive relative to airway pressure exacerbating obstruction
during expiration. Loop D demonstrates fixed airway
8 A
 recovered COVID-­19 patient who develops tracheal obstruction, as with tracheal stenosis, causing flatten-
stenosis following a 2 week intubation. ing of both components of the loop. Finally, loop E
demonstrates lower airway obstruction as seen in
COPD and asthma. A scooped-­out appearance to the
9 A
 75-­year-­old male who undergoes emergent intuba-
loop comes from premature airway closure as hetero-
tion following development of angioedema found to
geneity of flow in expiration, i.e., areas with higher
have R vocal cord paralysis.
elastic recoil and lower airway resistance empty faster
than diseased areas.
Flow volume loops involve plotting inspiratory and
expiratory flow on the Y-­ axis with volume on the Answers: 6-­E, 7-­B, 8-­C , 9-­B
X-­a xis, ideally during maximally forced inspiratory
and expiratory effort. Flow volume loops are compo- Loutfi SA and Stoller JK. Flow-­volume loops. UpToDate.
nent of the information presented on mechanical ven- Retrieved November 16, 2020 from https://www.uptodate.
tilators as well and can aid in the diagnosis of airway com/contents/flow-­volume-­loops?search=flow%20
obstruction. The normal loop is seen in loop A above volume%20loops&source=search_result&selected
representing a complete inspiratory and expiratory Title=1~59&usage_type=default&display_rank=1
breath. Loop B demonstrates variable extrathoracic Pellegrino R, Viegi G, Brusasco V, et al. Interpretative
obstruction with a flattening of the inspiratory compo- strategies for lung function tests. Eur Respir J. 2005;
nent. This is due to a combination of atmospheric 26(5): 948–968.
extraluminal pressure and negative intraluminal pres-
sure exacerbating extrathoracic obstruction as in vocal 10 A 72-­year-­old woman is admitted to the trauma
cord dysfunction and mobile tumors such as lipoma. ICU after presentation following high-­speed MVC.
6 Surgical Critical Care and Emergency Surgery

A pulmonary artery catheter is placed given the t­herapy may play a role in normotensive individuals.
patient’s refractory hypotension. Which of the fol- Vasopressors may be used in hypotensive patients with
lowing is consistent with cardiogenic shock? the goal of increasing systemic vascular resistance with-
out increasing pulmonary vascular resistance. Fluid
PCWP CO SVR MVO2 resuscitation should be adequate before continuing to
(mmHg) (L/min) (dyne-­sec/cm5) (%)
increase vasopressor use. The intra-­aortic balloon pump
A 8 5 1200 70 is used in left heart failure, not right heart failure.
B 4 3 1800 50 Answer: D
C 14 3 1800 50
D 8 8 1200 70 Ventetuolo CE and Klinger JR. Management of acute right
E 8 6 1800 70 ventricular failure in the intensive care unit. Ann Am
Thorac Soc. 2014; 11(5): 811–822.
Though used infrequently within the surgical ICU set-
ting, the Swan-­Ganz catheter is a useful adjunct in the 12 An 83-­year-­old woman with past medical history of
diagnosis of undifferentiated shock. Normal values significant peripheral vascular disease, ESRD on
obtained, as in option A, show a pulmonary capillary peritoneal dialysis admitted following below knee
wedge pressure (PCWP) 8–12 mmHg, cardiac output amputation for acute limb ischemia. You are called
(CO) 5–7 L/min, systemic vascular resistance (SVR) to bedside for patient’s mean arterial pressure of
900–1300dyne-­sec/cm5, and mixed venous oxygen 55 mmHg. You note the systolic pressure is appropri-
(MVO2) approximately 65%. Option B indicates severe ate, but diastolic pressure remains low. Which of the
hypovolemic shock with decreased PCWP, decreased following is part of the pathophysiology of diastolic
CO, increased SVR, and decreased MVO2. Option C heart failure?
indicates cardiogenic shock with increased PCWP, A Adaptive myocyte remodeling.
decreased CO, increased SVR, and decreased MVO2. B Volume overload of the ventricle.
Option D indicates distributive shock with normal C Cell loss secondary to increased oxygen demand.
PCWP, increased CO, decreased SVR, and increased D Impaired ventricular wall relaxation.
MVO2. Option E indicates obstructive shock with normal E Change of ventricle from elliptical to globular.
PCWP, normal CO, increased SVR, and increased MVO2.
Diastolic heart failure stems from incomplete relaxation
Answer: C of the ventricle. Three pathophysiologic pathways
include impaired ventricular wall relaxation, as left atrial
Cecconi M, De Backer D, Antonelli M, et al. Consensus on pressure exceeds left ventricular pressure causing pul-
circulatory shock and hemodynamic monitoring. Task monary edema; increased stiffness of the ventricle sec-
force of the European Society of Intensive Care ondary to increased wall thickness and decreased
Medicine. Intensive Care Med. 2014; 40: 1795–1815. internal diameter often seen with poorly controlled
hypertension; excess collagen deposition as myofibrils
11 A 73-­year-­old female with past medical history of sig- are laid in parallel secondary to ischemia, as seen with
nificant peripheral vascular disease, hypertension, MI, impairing contractility. The pathophysiology of sys-
and diabetes is admitted to the ICU with significant tolic failure involves adaptive myocyte remodeling, as
hypotension following a myocardial infarction in occurs with CAD, changing ventricular shape resulting
PACU after undergoing EVAR of a 6 cm AAA. STAT in an increasingly overloaded ventricle with decreasing
echocardiogram shows right-­sided heart failure. Swan-­ contractility resulting in cell loss due to increased oxygen
Ganz catheter is placed with PCWP of 10 mmHg. demand and eventual change of the ventricle from ellip-
What is the next appropriate intervention? tical to globular.
A Inotrope initiation.
B Vasopressor initiation. Answer: D
C Placement of intra-­aortic balloon pump.
D Volume resuscitation. Zile MR, Baicu CF and Gaasch WH. Diastolic heart
E Diuretic therapy. failure – abnormalities in active relaxation and passive
stiffness of the left ventricle. NEJM. 2004; 350:
The initial treatment of choice following acute right 1953–1959.
heart failure following MI is fluid resuscitation until
PCWP > 15 mmHg is reached. Following this, initiation 13 You are utilizing central venous pressure monitoring
of inotropes, such as dobutamine, is done. Diuretic to guide resuscitation of a patient with a 60% TBSA
Respiratory and Cardiovascular Physiology 7

burn injury in your ICU. Which of the following com- Answer: B


ponents of the CVP waveform represents isovolumic
contraction? Magder S. Central venous pressure: a useful but not so
A c wave simple measurement. Crit Care Med. 2006; 34:
B x descent 2224–2227.
C a wave
D y descent 15 Which of the following is NOT a physiologic effect of
E v wave minimally invasive left ventricular assist device?
A Decreased left ventricular end diastolic pressure.
a B Decreased left ventricular wall tension.
c
C Increased diastolic pressure.
D Increased mean arterial pressure.
v E Increased pulmonary capillary pressure.
x
y
A minimally invasive left ventricular assist device is a
miniature axial pump that allows blood to be aspirated
CVP Waveform from the left ventricle into the cannula component of the
component Mechanical event
pump and expelled above the aortic valve into the
a wave Atrial contraction ascending aorta. The device has been used for support in
c wave Isovolumic contraction high-­risk percutaneous coronary intervention as well as
v wave Systolic filling of the atrium cardiogenic shock. The device works to unload the left
x descent Atrial relaxation, systolic collapse ventricle reducing left ventricular end diastolic pressure
y descent Early diastolic filling, diastolic and wall tension. This allows for decreased oxygen
collapse demand. Furthermore, it increased mean arterial pres-
sure, diastolic pressure, and cardiac output, thereby
improving both systemic and coronary blood flow.
Answer: A Finally, it decreases pulmonary capillary pressure and
thereby right ventricular afterload.
Magder S. Central venous pressure: a useful but not so
simple measurement. Crit Care Med. 2006; 34: 2224–2227.
Answer: E
14 Which component of the cardiac cycle is represented
by the answer given above? Burzotta F, Trani C, Doshi S, et al. Impella ventricular
A Early diastole support in clinical practice: collaborative viewpoint from
B Early systole a European expert user group. Int J Cardiol. 2015; 201:
C End diastole 684–691.
D Mid-­systole
E End systole 16 Which of the following is a physiologic impact of
intra-­aortic balloon pumps during systole?
The a wave occurs from right atrial contraction increasing A Increased systolic blood pressure.
venous pressure. Right ventricular contraction displaces B Decreased pre-­systolic aortic pressure.
the tricuspid valve into the right atrium, represented by C Increase in the isometric phase of left ventricular
the c wave. With the emptying of right ventricle, the right contraction.
atrium relaxes and begins to fill, represented by the x D Increased left ventricular wall tension.
descent. The v wave demonstrates the filled right atrium E Decreased left ventricular ejection fraction.
with increased atrial pressure. Finally, the y descent shows
right ventricular filling as the tricuspid valve opens. The intra-­aortic balloon pump follows the principle of
counterpulsation i.e. inflation during diastole with defla-
CVP Waveform component Cardiac cycle event tion during systole. The physiologic impacts during the
systolic phase include a decrease in aortic systolic pres-
a wave End diastole
sure as well pre-­systolic (end-­diastolic) aortic pressure
c wave Early systole
both of which contribute to decreased afterload by 10%
v wave Late systole
and 30%, respectively; decrease in the isometric phase of
x descent Mid-­systole
left ventricular contraction, thereby reducing myocardial
y descent Early diastole
oxygen consumption; decreased left ventricular wall
8 Surgical Critical Care and Emergency Surgery

t­ension by 20%; increased left ventricular ejection frac- A Increased venous return.
tion by up to 30%. B Increased aortic pressure.
C Baroreceptor dampening.
Answer: B
D Increased systemic vascular resistance.
Parissis H, Graham V, Lampridis S, et al. IABP: history-­ E Increased preload.
evolution-­pathophysiology-­indications: what we need to With positive pressure ventilation, increased intrapleural
know. J Cardiothorac Surg. 2016; 11(1): 122. pressure results in initially increased aortic pressure
causing compensatory reduction in systemic vascular
17 Which of the following is an expected cardiovascular resistance and left ventricular afterload by activated
change during pregnancy? baroreceptors, thereby increasing cardiac output.
A Decreased heart rate. Positive pressure additionally decreases venous return
B Decreased cardiac output. and, therefore, preload.
C Increased peripheral vascular resistance.
D Decreased ventricular distension.
E Decreased systemic vascular resistance. Answer: B

Alviar CL, Miller PE, McAreavey D, et al. Positive pressure


Pregnancy results in increased heart rate, increased car-
ventilation in the cardiac intensive care unit. J Am Coll
diac output, decreased peripheral vascular resistance,
Cardiol. 2018; 72: 1532–1553.
increased ventricular distension, and decreased systemic
vascular resistance.
20 A 70-­year-­old woman in a motor vehicle collision
Answer: E undergoes a splenectomy for Grade IV laceration
and receives four units of whole blood in the OR but
Hill CC and Pickinpaugh J. Physiologic changes in preg- arrives in the ICU tachycardiac and hypotensive.
nancy. Surg Clin N Am. 2008; 88: 391–401. Point of care hemoglobin is 14.3 mg/dL 2 hours post-­
transfusion. Her abdomen was left open and mini-
18 Which of the following is a mechanism by which vas- mal output is coming from her negative pressure
odilators improve cardiac function in acute decom- abdominal dressing. She has multiple rib fractures
pensated heart failure? and a radius fracture. Which of the following thera-
A Increased ventricular preload. pies would promote end-­organ perfusion?
B Decreased stroke volume. A Decrease vasoactive drug doses (decrease periph-
C Increased ventricular afterload. eral vascular resistance).
D Increased cardiac output. B Increase sedation and pain medications to
E Increased ventricular filling pressure. decrease her heart rate.
C Increase end-­ diastolic volume with volume
The pathophysiology of acute heart failure involves resuscitation.
increased myocardial oxygen demand with increased ven- D Increase contractility with positive inotrope.
tricular filling pressures, low cardiac output, and increased E Increase end-­systolic volume.
systemic vascular resistance. Nitroprusside and nitroglyc-
erin remain two of the most potent vasodilators used in This patient has evidence of blunt chest trauma with mul-
therapy. Nitrogylcerin is a venodilator working to decrease tiple rib fractures and tachycardia. While she could have
preload, decrease afterload, and myocardial oxygen hypovolemic shock from her splenic injury and intraop-
demand. Nitroprusside is an arterial and venous dilator erative blood loss, she remains hypotensive despite trans-
decreasing preload, afterload, myocardial oxygen demand fusions with a hemoglobin of 14.3 mg/dL making this less
as well as increasing stroke volume and cardiac output. likely and no evidence of ongoing bleeding from her abdo-
Answer: D men. This makes it less likely that further volume resusci-
tation with blood or crystalloid would be helpful. Blunt
Carlson MD and Eckman PM. Review of vasodilators in cardiac injury can occur with blunt chest trauma and is
acute decompensated heart failure: the old and new. J initially screened for with EKG and troponin assessment,
Card Fail 2013; 19(7): 478–493. followed by an echocardiogram. Blunt cardiac injury may
be improved with positive inotropic medications.
19 Which of the following is an expected effect of Decreasing vasoactive drug doses would worsen
increased intrapleural pressure from positive pres- hypotension and worsen end-­ organ perfusion.
sure ventilation? Vasopressors are often used in supportive treatment for
Respiratory and Cardiovascular Physiology 9

blunt cardiac injury and may need to be increased to A-­a gradient equals PAO2−PaO2. His PaO2 from the
promote end-­organ perfusion. Increasing sedation and ABG is 50. The PAO2 can be calculated from this
pain medications may improve her tachycardia but equation:
would worsen her hypotension and end-­organ perfu- PaO2 FiO2 PB PH2 0 PaCO2 / RQ
sion. Increasing end-­systolic volume would decrease her
0.21 760 47 30 / 0.8
stroke volume and cardiac output further, worsening
her end-­organ perfusion. PaO2 112.5 mm Hg
Remember: CO HR SV
SV EDV ESV Therefore, A-­a gradient (PaO2−PAO2) = 62.5 mm Hg.
Answer: D Answer: E

Levick JR. An Introduction to Cardiovascular Physiology. Marino P. The ICU Book, 3rd ed., Lippincott Williams &
Butterworth and Co., London, 2013. Wilkins, Philadelphia, PA, chapter 19 2007.
Clancy K, Velopulos C, Bilaniuk JW, et al. Screening for
blunt cardiac injury: An Eastern Association for the 22 The patient above is placed on a nonrebreather
Surgery of Trauma practice management guideline. J mask with minimal improvement. What is the most
Trauma Acute Care Surg. 2012; 735: S301–S306. likely etiology of the above patient’s respiratory fail-
ure and appropriate intervention?
21 A 39-­year-­old man presents with a cold right leg and A Pulmonary embolism, anticoagulation.
complains of nine days of symptoms. Following a B Hyperventilation from anxiety, benzothiazines.
thromboembolectomy and fasciotomy, he develops C COVID-­ 19 pneumonia, dexamethasone, and
hypoxia with saturation of 87% and respiratory dis- high-­f low nasal canula.
tress. An arterial blood gas shows: pH 7.47, D Neuromuscular weakness, reversal of paralytic.
paO2 = 50 mm Hg, HCO3 = 22 mmol/L, E Pulmonary edema, acute kidney injury from
pCO2 = 30 mm Hg. Chest x-­ray shows patchy consoli- rhabdomyolysis.
dations bilaterally and he reports fever prior to
admission and that he works in a skilled nursing Hypoxemia occurs in four conditions: low inspired oxy-
facility during the pandemic. gen, shunt, V/Q mismatch, and hypoventilation.
Hypoventilation would present with high CO2 and
normal A-­a gradient. This could occur with overseda-
tion, neuromuscular weakness, and residual anesthesia.
Hyperventilation would cause tachypnea, low CO2, but
not hypoxia, so A-­a gradient should be normal. Low
inspired oxygen should have a low PO2 and normal gra-
dient. An acute PE or asthma exacerbation presents
with V/Q mismatch with elevated A-­a gradient and nor-
mal PCO2. It should correct with administration of oxy-
gen. Shunting (pulmonary edema or pneumonia) has an
elevated A-­a gradient that does not improve with oxy-
gen administration. The patient is young for postopera-
tive MI and has risk factors and a chest x-­ray consistent
with COVID-­19 pneumonia, which could also increase
his risk of thrombotic events since as an arterial
thrombus.

Answer: C

Based on the above results, his A-­a gradient is (at sea Weinberger SE, Cockrill BA and Mande J. Principles of
level, water vapor pressure = 47 mm Hg): Pulmonary Medicine, 5th ed., W.B. Saunders,
A 150 mm Hg Philadelphia, PA, (2008).
B 10 mm Hg NIH COVID-­19 Treatment Guidelines. Therapeutic
C 38 mm Hg management of patients with COVID-­19. www.
D 50 mm Hg covid19treatmentguidelines.nih.gov/therapeutic-­
E 62 mm Hg management/ (accessed 15 December 20).
10 Surgical Critical Care and Emergency Surgery

23 A 63-­year-­old patient with history of hypertension tion, low inspiratory pressures with plateau pressures
and type 2 diabetes presents with acute respiratory <30 cm H20, high PEEP levels are better than low PEEP
distress syndrome from pneumococcal pneumonia levels, and prone positioning for at least 12-­hour periods
and is being managed by the ICU team for severe per day with improved mortality. Less than 6 hours of
ARDS. After appropriate sedation and analgesia, prone position per day would not be recommended as it
which of the following is NOT an appropriate strat- is too short a time period.
egy for management?
A Low tidal volume ventilation (4–8ml/kg IBW). Answer: B
B Prone positioning <6 hours/day.
C Use of recruitment maneuvers. Fan E., Del Sorbo L, Goligher EC, et al. An Official
D Higher PEEP levels with plateau pressures <30 cm American Thoracic Society/European Society of
H2O. Intensive Care Medicine/Society of critical care medi-
E Very select use of high-­ frequency oscillatory cine clinical practice guideline: mechanical ventilation in
ventilation. adult patients with acute respiratory distress syndrome.
Am J Respir Crit Care Med. 2017; 195 9: 1253–1263.
Acute respiratory distress syndrome management guide- https://www.thoracic.org/statements/resources/cc/
lines target management with low tidal volume ventila- ards-­guidelines.pdf.
11

Cardiopulmonary Resuscitation, Oxygen Delivery and Shock


Kevin W. Cahill, MD, Harsh Desai, MD, and Luis Cardenas, DO, PhD
Department of Surgery, Christiana Care Health Care System, Newark, DE, USA

1 A
 72-­year-­old woman with a history of Child’s B cir- agent, and prolongs phase 3 of the cardiac action poten-
rhosis and supraventricular tachycardia is in the ICU tial. Amiodarone slows conduction rate and prolongs the
following laparotomy for strangulated ventral hernia. refractory period of the SA and AV nodes. It also pro-
She begins to complain of rapid heartbeat and is noted longs the refractory periods of the ventricles, bundles of
to be in an irregular, wide-­complex ventricular tachy- His, and the Purkinje fibers without exhibiting any effects
cardia on EKG. She maintains pulse and adequate on the conduction rate. Serious side effects include inter-
blood pressure. Which of the following is the best ini- stitial lung disease and liver dysfunction with elevated
tial therapy to administer? liver enzymes.
A Synchronized cardioversion.
B Adenosine 6 mg IV. Answer: C
C Amiodarone 150 mg IV.
D Defibrillation. Littmann L, Olson EG,Gibbs MA. Initial evaluation and
E Vagal maneuvers. management of wide-­complex tachycardia: a simplified
and practical approach. Am J Emerg Med. 2019; 37:
The 2020 ACLS guidelines differentiate between regular 1340–1345.
and irregular wide-­complex tachycardia with and with- Panchal AR, Bartos JA, Cabanas JG et al. Part 3: Adult
out pulse. In this instance, the patient is in an irregular basic and advanced cardiac life support: 2020 American
wide-­complex tachycardia, symptomatic, but stable as Heart Association guidelines for cardiopulmonary
evidence by pulse and pressure. Given this hemodynamic resuscitation and emergency cardiovascular care.
stability, synchronized cardioversion and defibrillation Circulation. 2020; 142 (suppl 2): S366–S468.
are not the initial therapies (choices A, D). Adenosine and
vagal maneuvers may be effective in regular ventricular 2 W
 hich of the following techniques has not been shown
tachycardia (choices B, E). Therefore, amiodarone is the to be effective in airway management during cardiac
best initial medication to administration often followed arrest?
by infusion (choice C). Individuals with hemodynami- A Head tilt – chin lift
cally unstable ventricular tachycardia should not initially B Jaw thrust
receive amiodarone. These individuals should be cardio- C Cricoid pressure
verted. Amiodarone can be used regardless of the indi- D Nasopharyngeal airway
vidual’s underlying heart function and the type of E Oropharyngeal airway
ventricular tachycardia. It can be used in individuals with
monomorphic ventricular tachycardia, but is contraindi- Of the above maneuvers, cricoid pressure has not been
cated in individuals with polymorphic ventricular tachy- shown to be effective during airway management in car-
cardia as it is associated with prolonged QT intervals, diopulmonary resuscitation. It may impede ventilation
which will be made worse with anti-­arrhythmic drugs. or placement of airway adjuncts such as a supraglottic
Amiodarone is categorized as a class III ­anti-­arrhythmic airway as well as contribute to increased airway trauma.

Surgical Critical Care and Emergency Surgery: Clinical Questions and Answers, Third Edition.
Edited by Forrest “Dell” Moore, Peter M. Rhee, and Carlos J. Rodriguez.
© 2022 John Wiley & Sons Ltd. Published 2022 by John Wiley & Sons Ltd.
Companion website: www.wiley.com/go/surgicalcriticalcare3e
12 Surgical Critical Care and Emergency Surgery

Jaw thrust is preferred in patients with suspected spinal 4 W


 hich of the following is the minimum chest compres-
injury. Nasopharyngeal and oropharyngeal airways are sion fraction (defined as amount of time spent deliver-
particularly useful in cases of facial trauma though care ing chest compressions during CPR) shown to be
must be taken with possible basilar skull fractures. associated with improved survival?
A 0–20%
Answer: C
B 21–40%
Carauna E, Chevret S, Pirracchio R. Effect of cricoid C 41–60%
pressure on laryngeal view during prehospital tracheal D 61–80%
intubation: a propensity-­based analysis. Emerg Med J. E 81–100%
2017; 34 (3): 132–137.
Panchal AR, Bartos JA, Cabanas JG et al. Part 3: Adult Optimal outcomes have been demonstrated with mini-
basic and advanced cardiac life support: 2020 American mal pauses between compressions for pulse checks and
Heart Association guidelines for cardiopulmonary breaths given during high-­quality CPR. A compression
resuscitation and emergency cardiovascular care. fraction of at least 60% has been shown to be necessary
Circulation. 2020; 142 (suppl 2): S366–S468. for best outcomes. Animal studies previously conducted
have demonstrated decreased coronary and cerebral
3 I n a patient experiencing PEA arrest, which of the fol- perfusion when chest compressions are not being con-
lowing would not be a likely etiology? ducted resulting in worsened outcomes. Multiple retro-
A Hypoglycemia spective analyses and cohort studies have resulted in
B Hypoxia many emergency agencies targeting a compression frac-
C Hypovolemia tion of between 60 and 80% as a quality metric. This
D Hypokalemia involves delivery of high-­quality compressions of appro-
E Hypocalcemia priate depth, 2 inches, and rate, at least 100/min.
50% Final vital status by CCF category
Pulseless electrical activity is so named due to evidence
of cardiac mechanical activity on echocardiogram or
rhythm on EKG. The algorithm is similar to the asystole
algorithm utilizing compressions and epinephrine. The
traditional etiologies are described as “Hs” and “Ts.” The 40%

“Hs” include hypoglycemia, hypoxia, hyper/hypoka- 29%


25%
lemia, hypovolemia, acidosis, and hypothermia.
23% 25%
Hypocalcemia can present with muscular and neuro-
logic symptoms such as perioral numbness, cramping,
Percentage surviving

30%
fatigue, seizures, and irritability. Hypocalcemia may also
be associated with increased risk of arrhythmias, but is
not typically considered high on the initial differential of
PEA arrest. The “Ts” taught as etiologies include tension
pneumothorax, cardiac tamponade, toxins, pulmonary 20%
12%
thrombosis, or coronary thrombosis. Evaluation for
pneumothorax or tamponade includes rapid bedside
physical exam as well as point of care ultrasound for rule
out. Ultrasound may also reveal signs of thrombosis with 0%
right ventricular enlargement or free-­floating thrombus.
Answer: E

Andersen LW, Holmberg MJ, Berg KM et al. In hospital 0%


cardiac arrest: a review. JAMA. 2019; 321 (12): 0% – 20% 21% – 40% 41% – 60% 61% – 80% 81% – 100%
n = 100 n = 74 n = 117 n = 143 n = 72
1200–1210.
Panchal AR, Bartos JA, Cabanas JG et al. Part 3: Adult CCF category

basic and advanced cardiac life support: 2020 American Answer: D


Heart Association guidelines for cardiopulmonary
resuscitation and emergency cardiovascular care. Christenson J, Andrusiek D, Everson-­Stewart S et al. Chest
Circulation. 2020; 142 (suppl 2): S366–S468. compression fraction determines survival in patients
Cardiopulmonary Resuscitation, Oxygen Delivery and Shock 13

with out of hospital ventricular fibrillation. Circulation. If a cardiac cause is suspected, pursuit of cardiac inter-
2009; 120: 1241–1247. vention such as with percutaneous coronary interven-
Panchal AR, Bartos JA, Cabanas JG et al. Part 3: Adult tion (PCI) is strongly recommended. Hyperoxygenation
basic and advanced cardiac life support: 2020 American therapy, the use of corticosteroids, and seizure prophy-
Heart Association guidelines for cardiopulmonary laxis have thus far shown no survival benefit (choices A,
resuscitation and emergency cardiovascular care. C, and E). Finally, targeted temperature management is
Circulation. 2020; 142 (suppl 2): S366–S468. currently recommended for post-­arrest care with target
of 32–36°C. This is based on several studies showing
5 W
 hich of the following is considered the highest pre- potential neurologic benefit. Preventing fever has not yet
dictor of survival for in-­and out-­of-­hospital CPR? been proven to improve outcome though the 2020 AHA
A Age. guideline (choice D). Ischemic heart disease is a major
B Shockable rhythm. cause of out of hospital cardiac arrest. Among patients
C Arrest at home. who had been successfully resuscitated after out of hos-
D Arrest at night vs during the day. pital cardiac arrest and had no signs of STEMI, immedi-
E Delayed EMS response time. ate angiography was not found to be better than a
strategy of delayed angiography with respect to overall
On the whole, survivability is dependent on patient, system, survival at 90 days.
event, and therapeutic factors. With increasing comorbid-
ity and age, survivability decreases. System factors include Answer: B
time to arrival of EMS, time to initiation of CPR, and time
Panchal AR, Bartos JA, Cabanas JG et al. Part 3: Adult
to defibrillation. Event factors include preceding symptoms.
basic and advanced cardiac life support: 2020 American
Finally, therapeutic factors include availability of medica-
Heart Association guidelines for cardiopulmonary
tions to treat suspected cause, time to ER, time to cath lab
resuscitation and emergency cardiovascular care.
should it be required, etc. The greatest mortality risk with
Circulation. 2020; 142 (suppl 2): S366–S468.
out of hospital cardiac arrest stems from unwitnessed
Yannapoulos D, Bartos JA, Aufderheide TP et al. The
arrests without bystander CPR often occurring at night in
evolving role of the cardiac catherization laboratory in the
the elderly. Highest survivability stems from witnessed
management of patients with out of hospital cardiac
arrests with rapid initiation of bystander CPR and initial
arrest: a scientific statement from the American Heart
shockable rhythm, such as ventricular fibrillation.
Association. Circulation. 2019; 139 (12): e530–e552.
Answer: B Lemkes JS, Janssens GN, van der Hoeven NW et al.
Coronary angiography after cardiac arrest without
Myat A, Song K-­J, Rea T. Out of hospital cardiac arrest: ST-­Segment elevation. April 11, 2019. N Engl J Med.
current concepts. Lancet. 2018; 391: 970–79. 2019; 380: 1397–1407. DOI: https://doi.org/10.1056/
Navab E, Esmaelli M, Poorkhorshidi N et al. Predictors of NEJMoa1816897
out of hospital cardiac arrest outcomes in pre-­hospital
settings; a retrospective cross-­sectional study. Arch Am 7 A
 35-­year-­old, 26 week pregnant woman has cardiac
Emerg Med. 2019; 7 (1): e36. arrest with CPR ongoing in the ED. CPR has been
ongoing for 5 minutes. Which of the following has been
6 A
 70-­year-­old man is 2 weeks status-­post laparo- shown to provide greatest benefit for achieving ROSC?
scopic sleeve gastrectomy and he undergoes witnessed A Corticosteroids.
cardiac arrest at home after complaint of new onset B Targeted temperature management.
chest pain. Bystander CPR achieves ROSC after C Left lateral uterine displacement.
10 minutes. He is now in the ICU, intubated, and on D Fetal monitoring.
vasopressors for associated hypotension. Which of the E C-­section.
following interventions has the strongest associated
survival benefit in post-­arrest care according to cur- In conditions of cardiac arrest after pregnancy, rapid
rent resuscitation guidelines? delivery of the fetus, typically by C-­section, termed peri-
A Maintain 100% FiO2. mortem cesarean delivery (PMCD), has been shown to
B Pursuit of cardiac intervention when STEMI be associated with improved outcomes when CPR does
identified. not achieve ROSC. However, the decision must be made
C Use of corticosteroids. quickly as a review article states that if done within
D Targeted temperature management to prevent fever. 10 minutes of arrest, it was associated with better mater-
E Seizure prophylaxis. nal outcomes. It was also thought that it was beneficial to
14 Surgical Critical Care and Emergency Surgery

the mother in 31% of cases and was not harmful in any was hypothermic or alkalotic, these conditions would
case. The review of the cases resulted in only 94 cases also shift it toward the left.
supporting that PMCD is rare. Corticosteroids have
shown no benefit and targeted temperature manage- 100
ment may be used after achievement of ROSC (choices A
and B).The left lateral uterine displacement alleviates
aortocaval compression in patients with hypotension, 80

Oxygen saturation of hemoglobin (percent)


Left-shifted
but delivery achieves this much more effectively (choice
C). Fetal monitoring during maternal CPR is a distrac-
tion and may hinder care (choice D). 60
Right-shifted
Answer: E

Einav S, Kaufman N, Sela HY. Maternal cardiac arrest and 40


perimortem caesarean delivery: evidence or expert
based? Resuscitation. 2012; 83 (10): 1191–1200.
Panchal AR, Bartos JA, Cabanas JG et al. Part 3: Adult 20
basic and advanced cardiac life support: 2020 American
Heart Association guidelines for cardiopulmonary
resuscitation and emergency cardiovascular care. 0
Circulation. 2020; 142 (suppl 2): S366–S468. 0 20 40 60 80 100
Partial pressure of oxygen (mmHg)
8 W
 hich of the following scenarios causes a shift of the
oxygen dissociation curve to the left? Answer: A
A A patient found unconscious in a basement apart- Woodson, RD. Physiologic significance of oxygen dissocia-
ment with malfunctioning heater. tion curve shifts. Crit Care Med. 1979; 7 (9): 368–373.
B Patient with pneumonia and fever of 102°C.
C Patient with lactic acidosis from mesenteric 9 Y
 ou are caring for a patient in the SICU, currently
ischemia. intubated after undergoing left upper lobectomy for
D Patient with depressed mental status taking slow, tumor. Patient’s current hemoglobin is 10 g/dL, oxygen
shallow breaths. saturation 95%, and PaO2 of 92 mmHg. What is the
E Patient returning from climbing Mt Everest where expected oxygen content (CaO2)?
he had to stop and be treated for hypoxia after leav- A 0.9 mL/dL
ing base camp. B 9 mL/dL
C 13 mL/dL
Everest where he had to stop and be treated for hypoxia D 21 mL/dL
after leaving base camp. The oxygen–hemoglobin disso- E 140 mL/dL
ciation curve is sigmoidal in shape based on allosteric
interactions of each globin monomer binding oxygen. A Blood oxygen content is based on the following formula
shift to the right indicates decreased affinity favoring influenced by oxygen saturation, partial pressure of arte-
unloading of oxygen while a shift to the left achieves the rial oxygen, and patient’s hemoglobin:
opposite effect. The strength by which oxygen binds to
CaO2 1.34 Hb SaO2 0.003 PaO2
hemoglobin is affected by several factors and can be rep-
resented as a shift to the left or right in the oxygen dis- 1.34 10 0.95 0.003 92
sociation curve. A rightward shift of the curve indicates 12.73 0.28
that hemoglobin has a decreased affinity for oxygen,
13.01 or 13 mL / dL
thus, oxygen actively unloads. A shift to the left indicates
increased hemoglobin affinity for oxygen and an The single biggest factor for oxygen content is hemo-
increased reluctance to release oxygen. Several physio- globin. Doubling of hemoglobin would double the oxy-
logic factors are responsible for shifting the curve left or gen content. Increasing the partial pressure of oxygen
right, such as pH, carbon dioxide (CO2), temperature, from 60 mmHg to 100 would increase saturation from
and 2,3-­Disphosphoglycerate. Carbon monoxide expo- 90 to 100% and would not be a large change in content.
sure, as can be seen in enclosed spaces with a malfunc- The doubling of partial pressure of oxygen from
tioning heater, can result in a leftward shift. If the patient 60 mmHg to 120 mmHg would still only increase the
Cardiopulmonary Resuscitation, Oxygen Delivery and Shock 15

content by 10% as the dissolved amount of oxygen in intervention, what would be the best next step to help
plasma is negated by the factor of 0.003. The constant confirm the likely diagnosis?
of 1.34 is the amount of oxygen that one gram of hemo- A Chest X-­ray
globin carries at 1 atmosphere of pressure. B CT angiogram
C CBC
Answer: C
D EKG
Crocetti J, Diaz-­Abad M, Krachman SL. Oxygen content, E Transthoracic echocardiogram
delivery, and uptake. In GJ Criner, RE Barnette, GE
D’Alonzo (Eds), Critical Care Study Guide. New York: This patient is exhibiting signs of cardiac tamponade,
Springer, 2010. with evidence of pulsus paradoxus, jugular venous dis-
tension, and hypotension. The primary tool for diagnosis
10 Changes in which of the following components is the of cardiac tamponade is Doppler echocardiography,
most influential in increasing oxygen delivery? which in the presence of tamponade typically shows a
A Cardiac output. circumferential pericardial fluid layer and compressed
B Hemoglobin level. chambers with high ventricular ejection fractions. On
C Oxygen saturation. inspiration, both the ventricular and atrial septa move
D Oxygen dissolved in blood. leftward and reverse on expiration, due to the fixed peri-
E Systemic vascular resistance. cardial volume. Right ventricular collapse is typically less
sensitive but more specific for tamponade. The inferior
As described in the question above, oxygen content is vena cava is typically dilated with minimal respiratory
influenced by hemoglobin, oxygen saturation, and par- variation. CT angiogram may demonstrate pericardial
tial pressure of arterial oxygen. Of these, hemoglobin effusion, distension of the superior and inferior vena
level, which has the greatest impact on oxygen content cavae, and reflux of contrast material into the azygos
through binding, has the greatest impact on oxygen vein and inferior vena cava. However, these represent
available to deliver to tissues. Arterial oxygen saturation static images rather than the dynamic information pre-
and cardiac output are additional important factors in sented by echocardiography. Chest x-­ray may demon-
ensuring adequate oxygen delivery. Increased cardiac strate an enlarged cardiac silhouette but is particularly
output as a compensatory mechanism can carry more unreliable in early/acute tamponade (choice A).
oxygenated blood for delivery. Improved oxygen satura- Additionally, obtaining a CT scan is typically not porta-
tions ensure appropriate oxygen availability for hemo- ble, requiring transporting a hemodynamically unstable
globin binding. Changes in vascular resistance can patient to obtain the study (choice B). A CBC would be of
influence oxygen diffusion. The least influential of the little use to obtaining this diagnosis (choice C). EKG may
above choices given, the minimal contribution it makes show evidence of pericarditis or electrical alternans but
to available oxygen, is partial pressure of arterial oxygen is unreliable in the diagnosis of tamponade (choice D).
i.e. dissolved oxygen. Answer: E
Answer: B
Spodick DH. Acute cardiac tamponade. N Engl J Med.
2003; 349 (7): 684–90. doi: https://doi.org/10.1056/
Marino P. The ICU Book, 4th edn. Philadelphia: Lippincott NEJMra022643. PMID: 12917306.
Williams & Wilkins, 2007.
12 A 27-­year-­old man presents after jumping from a
11 Y
 ou are called to the PACU to evaluate a 64-­year-­old diving board and striking the bottom of a pool with
man with a history of metastatic lung cancer now s/p his upper body. On presentation, he has no sensation
video-­ assisted thoracoscopic resection of the left or motor strength of his lower extremities. On exami-
upper lobe. His heart rate is 110 beats/min, blood nation, he appears flaccid and you cannot elicit spinal
pressure 70/42 mm Hg. He appears tachypneic. On reflexes. His heart rate is 54 beats/min, blood pressure
examination, he is cool and clammy, with evidence 90/54, and respiratory rate 18. Despite appropriate
of peripheral cyanosis and prominent jugular venous fluid resuscitation, he remains hypotensive, though
distension. Anesthesia has successfully placed an you identify no evidence of ongoing hemorrhage. What
arterial line and initiated several fluid boluses while type of shock does this likely represent?
awaiting your arrival; however, there has been no A Obstructive
significant improvement in his hemodynamics. You B Distributive
note that his systolic blood pressure on the arterial C Cardiogenic
line appears to decrease by at least 10 mmHg during D Hypovolemic
respiration. While you prepare the appropriate E Anaphylactic
16 Surgical Critical Care and Emergency Surgery

This patient demonstrates bradycardia, hypotension, 13 Shock is defined as:


and neurologic deficits in the setting of possible cervical A Blood pressure less than 90 mm Hg.
or high thoracic spine trauma, suggesting he may have a B Heart rate greater than 140 beats/min.
component of neurogenic shock. This shock is a result of C Urine output less than 0.5 ml/kg/hr.
spinal cord injury with sudden loss of sympathetic tone D Inadequate perfusion to meet end organ meta-
with preserved parasympathetic activity and autonomic bolic needs.
instability, leading to bradycardia and hypotension. E All of the above.
These changes are typically seen with an injury to the
spinal cord above T6. Disruption of the sympathetic divi- Shock is defined by some as inadequate perfusion to
sion of the autonomic nervous system affects three areas meet end organ metabolic needs. Tissue and cellular
of the cardiovascular system: coronary blood flow, car- hypoxia can be due to inadequate delivery, increased
diac contractility, and heart rate. There is systemic hypo- consumption, inadequate utilization, or a combination
tension due to a decrease in sympathetic fiber-­mediated of these states. Although this is often reflected in hemo-
arterial and venous vascular resistance, along with dynamic changes such as hypotension, tachycardia, or
venous pooling and loss of preload, with or without oliguria, these are not sufficient criteria alone to diagno-
bradycardia. The bradycardia is often exacerbated by sis a patient as being in shock. A patient may present
suctioning, defecation, turning, and hypoxia. The hypo- hypertensive, normotensive, or hypotensive. Conditions
tension places patients at increased risk of secondary such as neurogenic shock may result in a patient with
spinal cord ischemia due to impairment of autoregula- bradycardia despite inadequate perfusion. Shock can be
tion. With preserved parasympathetic activity, this trans- further differentiated into hypovolemic, cardiogenic,
lates clinically into bradycardia (and possibly other obstructive, or restrictive (vasodilatory/distributive).
cardiac arrhythmias) in the setting of profound hypoten- Causes of obstructive shock include pulmonary embo-
sion. Trauma patients are hypotensive as a result of blood lism, tension pneumothorax, and pericardial tampon-
loss or intravascular hypovolemia but will mount an ade. Causes of obstructive shock typically lead to
appropriate tachycardic response. Blood loss must be decreased cardiac output and are sometimes included
ruled out and treated appropriately before assuming that into the cardiogenic shock category. Identification of
hypotension is due solely to spinal cord injury. It is com- these sub categories of shock is crucial to guiding thera-
mon to have both blood loss and spinal cord injury. peutic intervention.
Initial management is composed of volume resuscitation
to account for the increased intravascular space second-
ary to increased vasodilation, as well as vasopressors for
blood pressure control. In addition to pressor support, Cardiogenic shock

chronotropic and inotropic support may be necessary. Extrinsic (tamponade)

Norepinephrine is started initially but in refractory


cases, epinephrine and vasopressin infusions may be Intrinsic (failure, ischemia)

required. Bradycardia usually responds to atropine and


glycopyrrolate but in severe cases, dopamine infusion is
required. When blood loss is a part of the presentation, Hemorrhagic
volume resuscitation should be with blood products and
not crystalloids. Spinal shock is often confused with neu-
rogenic shock. Spinal shock, on the other hand, refers to
loss of all sensation below the level of injury and is not
circulatory in nature. Both may, however, coexist in a
patient. Distributive

Neurogenic
Answer: B

Stein DM, Knight WA. Emergency neurological life


support: traumatic spine injury. Neurocrit Care. 2017; 27 Answer: D
(Suppl 1): 170–180.
Phillips AA, Krassioukov AV. Contemporary cardiovascu- Kislitsina ON, Rich JD, Wilcox JE et al. Shock -­classifica-
lar concerns after spinal cord injury: mechanisms, tion and pathophysiological principles of therapeutics.
maladaptations, and management. J Neurotrauma. 2015; Curr Cardiol Rev. 2019; 15 (2): 102–113. doi: https://doi.
32 (24): 1927–42. doi: https://doi.org/10.1089/ org/10.2174/1573403X15666181212125024. PMID:
neu.2015.3903. Epub 2015 Sep 1. PMID: 25962761. 30543176; PMCID: PMC6520577.
Cardiopulmonary Resuscitation, Oxygen Delivery and Shock 17

Vincent JL, De Backer D. Circulatory shock. N Engl J Med. an­


esthesiologist immediately notes a marked
2013; 369: 1726. decrease in the patient’s end-­tidal carbon dioxide
and oxygen saturations as well as new onset tachy-
14 A 53-­year-­old woman with a history of ulcerative cardia. You halt insufflation but the patient quickly
colitis controlled with 50 mg of oral prednisone daily becomes hemodynamically unstable. What is your
undergoes a laparoscopic converted to open colec- best step to address the underlying pathology?
tomy. Intra-­operatively there are no complications A Convert to open.
noted and she receives appropriate fluid resuscita- B Place the patient in steep Trendelenburg and
tion. However, post-­operatively she is noted to be place a central line for therapeutic intervention.
febrile and hypotensive. This hypotension is refrac- C Administer fluid bolus.
tory to additional fluid boluses or multiple vaso- D Start vasopressors.
pressors. On physical examination, her abdomen E Abort the procedure and transfer the patient to
does not appear distended and she is appropriately the ICU.
tender. What would be the best next step in manage-
ment of this patient? This patient is demonstrating evidence of possible air
A Additional fluid boluses. embolism secondary to intravascular insufflation. The
B Adding on an additional vasopressor. primary goal in this case is to prevent further gas entry
C Return to the operating room for exploration. into the venous system and reduce the amount of gas
D Administer stress dose hydrocortisone. trapped in the heart. Placing the patient in Trendelenburg
E Begin broad-­spectrum antibiotics. position maximizes blood flow to the brain and theoreti-
cally relieves right-­sided heart airlock as well as prevent
This patient with a history of chronic adrenal suppres- gas entry into the pulmonary artery. In a patient who is
sion due to daily prednisone use presents with signs and hemodynamically unstable secondary to an air embo-
symptoms consistent with an adrenal crisis. These events lism, a central line should be placed into the right atrium
are typically brought on by an inability for the body to and attempts made to withdraw air from the right side of
mount an appropriate response to an insult by generat- the heart. Converting to open would not address the
ing endogenous cortisol secondary to chronic adrenal underlying issue (choice A). Initiating fluids or vasopres-
suppression. Unless administered appropriate exoge- sors would briefly temporize the patient but would not
nous glucocorticoids, they may exhibit evidence of hypo- address the underlying pathology (choices C, D).
tension refractory to typical interventions, abdominal Aborting the procedure and taking a hemodynamically
pain, nausea/vomiting, and confusion. Additional fluid unstable patient to the ICU would not be correct as the
boluses or adding an additional vasopressor would not underlying pathology should be addressed prior to leav-
address the underlying pathology and has already been ing the operating room (choice E).
described as unsuccessful in this vignette (choices A, B).
Initiating broad-­spectrum antibiotics similarly does not Answer: B
address the underlying issue and would have no impact
on this patient’s hemodynamics (choice E). Septic shock Sandadi S, Johannigman JA, Wong VL et al. Recognition
would most likely develop later and not immediately. and management of major vessel injury during laparos-
While a hypotensive patient post-­operatively may be due copy. J Minim Invasive Gynecol. 2010; 17 (6): 692–702.
to blood loss and ultimately require return to the operat- doi: https://doi.org/10.1016/j.jmig.2010.06.005. Epub
ing room for exploration, in this case, failure to recog- 2010 Jul 24. PMID: 20656569.
nize the underlying adrenal crisis would result in
unnecessary re-­exploration (choice C). 16 A 32-­year-­old healthy man was passed out in a
Answer: D workplace fire but had minimal burns to the right
hand. Given suspected inhalation injury, you take
Rushworth RL, Torpy DJ, Falhammar H. Adrenal Crisis. N care to establish a definitive airway and transfer the
Engl J Med. 2019; 381 (9): 852–861. doi: https://doi. patient to the ICU for additional monitoring. The
org/10.1056/NEJMra1807486. PMID: 31461595. patient is initially tachycardic and hypertensive but
shortly thereafter develops bradycardia, hypoten-
15 A 71-­ year-­
old patient has acute, non-­
perforated sion, and cardiac dysrhythmias. On physical exami-
appendicitis. His BMI is 27 and otherwise healthy. nation, his skin appears flushed with a cherry-­red
Intra-­
operatively you begin with Veress needle color. Labwork reveals a marked metabolic acidosis
insertion into the abdomen and begin to establish on arterial blood gas and serum lactate is 9 mmol/L.
pneumoperitoneum with high flow rates. Your His carboxyhemoglobin level is normal. Which of the
18 Surgical Critical Care and Emergency Surgery

following would be most effective in addressing his fracture in this patient). The triad of hypoxemia, neuro-
underlying pathology? logic abnormalities, and petechial rash is classic for fat-­
A Aggressive fluid resuscitation embolism syndrome, though non-­specific. Fat embolism
B Administration of hydroxocobalamin can also present with thrombocytopenia and this may
C Vasopressor support help make a diagnosis. However, it remains a diagnosis of
D Diuresis exclusion, primarily made clinically. Initial assessment is
E Continue supportive care performed to exclude alternative diagnoses such as pul-
monary embolism. There is no definitive treatment and
This patient is showing evidence of possible cyanide poi- therapy is primarily supportive while awaiting resolu-
soning with evidence of cardiovascular instability, tion. There is no role for intravascular lytic therapy or
marked metabolic acidosis, and classic “cherry-­red” skin broad-­spectrum antibiotics (choices B, C). While vaso-
color. Although present in only a minority of patients, pressors and invasive ventilator support such as ECMO
this finding is a result of impaired tissue oxygen utiliza- may be necessary in patients with refractory shock, they
tion, resulting in high venous oxyhemoglobin concentra- are not the initial step in management (choices D, E).
tion, and bright red appearance of the blood.
Answer: A
Hydroxocobalamin is a precursor of Vitamin B12 that
directly binds to intra-­cellular cyanide, forming cyano- Stein PD, Yaekoub AY, Matta F et al. Fat embolism
cobalamin. This molecule is then readily excreted in the syndrome. Am J Med Sci. 2008; 336: 472.
urine. This treatment acts rapidly, does not affect tissue
oxygenation, and is relatively safe, making it a first-­line 18 A 54-­year-­old patient with a history of diabetes mel-
agent for cyanide poisoning. The other answer questions litus on home metformin presents to your emergency
do not address what is driving the patient’s underlying department with shortness of breath, productive
pathology. cough, and fever. On imaging, he is found to have a
Answer: B right lower lobe opacity consistent with pneumonia.
He is hemodynamically stable but blood work is
Hendry-­Hofer TB, Ng PC, Witeof AE et al. A review on noted to have a lactic acidemia of 4 and his glucose
ingested cyanide: risks, clinical presentation, diagnos- is elevated to 300. His CBC is within normal limits
tics, and treatment challenges. J Med Toxicol. 2019; 15: and an EKG is normal. He is mentating well, making
128. appropriate urine without evidence of tissue hypop-
erfusion. What best describes the patient’s lactic
17 A 37-­year-­old patient is admitted to the floor after academia?
suffering a femur fracture during a MVC. While he is A Type A lactic acidosis
stable over the next 24 hours, he shortly thereafter B Type B lactic acidosis
develops a new petechial rash on the non-­dependent C Septic shock
portions of his body, becomes hypotensive, confused, D Hemorrhagic shock
tachypneic, and is hypoxic on pulse oximetry. A chest E Cardiac failure
x-­ray is obtained but appears normal. A CT angio-
gram of the chest does not demonstrate any evidence This patient is showing evidence of lactic acidosis in the
of pulmonary thromboembolism. What would be the absence of systemic hypoperfusion. Type A lactic acido-
next step in management? sis is typically related to hypoperfusion secondary to
A Supportive care with fluid resuscitation and hypovolemia, cardiac failure, sepsis, or cardiopulmonary
oxygenation arrest. Type B lactic acidosis occurs when there is no evi-
B Intravascular tPA lytic therapy dence of systemic hypoperfusion and may be related to
C Broad-­spectrum antibiotics impaired cellular metabolism (choice B). Both met-
D Vasopressors formin use and diabetes mellitus have been implicated as
E ECMO associated with Type B lactic acidosis. This patient is
showing no signs of septic, hemorrhagic, or cardiogenic
This patient is showing evidence of possible fat-­embolism shock (choices C, D, E).
syndrome. This is a rare entity that can be encountered
in patients 24–72 hours after an initial insult (long bone Answer: B
19

ECMO
Mauer Biscotti III, MD1, Matthew A. Goldshore, MD, PhD, MPH2, and Jeremy W. Cannon, MD, SM3,4
1
Division of General Surgery, Department of Surgery, San Antonio Military Medical Center, San Antonio, TX, USA
2
Department of Surgery, Perelman School of Medicine at the University of Pennsylvania, Philadelphia, PA, USA
3
Division of Traumatology, Surgical Critical Care & Emergency Surgery, Perelman School of Medicine at the University of Pennsylvania, Philadelphia, PA, USA
4
Department of Surgery, F. Edward Hébert School of Medicine, Uniformed Services University of the Health Sciences, Bethesda, MD, USA

1 A 45-­year-­old previously healthy man was a pedes- of ventilator dyssynchrony. If the patient’s oxygenation
trian struck by a motor vehicle resulting in multiple does not improve, ECMO is reasonable so long as his
injuries including traumatic brain injury with a suba- traumatic brain injury is not severe, his intracranial
rachnoid hemorrhage (SAH), multiple rib fractures, bleeding has stabilized, and there is no ongoing torso
pulmonary contusion, hemothorax, splenic lacera- hemorrhage. The RESP score calculator can be used to
tion, and a pelvic fracture. On postinjury day 5, he quantify the patient’s projected outcome on ECMO
developed severe hypoxemic respiratory failure (https://www.elso.org/Resources/ECMOOutcome
(PaO2:FiO2 ratio of 70 on FiO2 of 1) and was diag- PredictionScores.aspx).
nosed with an MRSA pneumonia. Workup for other High-­frequency oscillatory ventilation requires special
causes of respiratory failure or sepsis was negative, expertise and does not offer any clear survival benefit for
and there was no evidence of SAH progression or torso this patient. Airway pressure release ventilation (APRV)
hemorrhage on his most recent imaging. Which of the is better suited to awake patients with moderate respira-
following should be performed before considering this tory failure and ventilator synchrony problems. Rib frac-
patient for extracorporeal membrane oxygenation ture stabilization should be performed earlier in the
(ECMO)? hospital course. The patient would not likely benefit
A High-­frequency oscillatory ventilation from this procedure and also would be unlikely to signifi-
B Airway pressure release ventilation cantly improve with this intervention. In the absence of
C Prone positioning abdominal compartment syndrome or refractory intrac-
D Rib fracture stabilization ranial pressure elevation, decompressive laparotomy has
E Decompressive laparotomy no role in the management of this patient.

This patient is potentially a good candidate for veno- Answer: C


venous extracorporeal membrane oxygenation (ECMO)
for hypoxemic respiratory failure. The basic principles Brodie D, Bacchetta M. Extracorporeal membrane
for determining a patient’s candidacy for ECMO include oxygenation for ARDS in adults. N Engl J Med.
lack of response to conventional ventilator management 2011;365(20):1905–14. doi: https://doi.org/10.1056/
and rescue interventions for severe hypoxemic or hyper- NEJMct1103720. PMID: 22087681.
carbic respiratory failure, an underlying process that is Brodie D, Slutsky AS, Combes A. Extracorporeal life
potentially reversible, and no contraindications to support for adults with respiratory failure and related
ECMO. The ventilator should be optimized for acute indications: a review. JAMA. 2019;322(6):557–568. doi:
respiratory distress syndrome (ARDS) management, and https://doi.org/10.1001/jama.2019.9302. PMID:
proning can be employed as a rescue intervention to 31408142.
optimize gas exchange. Chemical paralysis can also be Bullen EC, Teijeiro-­Paradis R, Fan E. How i select which
used along with deep sedation, particularly in the setting patients with ARDS should be treated with venovenous

Surgical Critical Care and Emergency Surgery: Clinical Questions and Answers, Third Edition.
Edited by Forrest “Dell” Moore, Peter M. Rhee, and Carlos J. Rodriguez.
© 2022 John Wiley & Sons Ltd. Published 2022 by John Wiley & Sons Ltd.
Companion website: www.wiley.com/go/surgicalcriticalcare3e
20 3 Surgical Critical Care and Emergency Surgery

extracorporeal membrane oxygenation. Chest. proven safe and effective. Obesity is no longer a con-
2020;158(3):1036–1045. doi: https://doi.org/10.1016/j. traindication to ECMO, and in select patients it may
chest.2020.04.016. Epub 2020 Apr 21. PMID: 32330459. even be protective. Severe aortic valve insufficiency is a
Cannon JW, Gutsche JT, Brodie D. Optimal strategies for relative contraindication to VA ECMO. Mild aortic valve
severe acute respiratory distress syndrome. Crit Care insufficiency may require venting of the left ventricle
Clin. 2017;33(2):259–275. doi: https://doi.org/10.1016/j. with a microaxial pump, atrial septostomy, or LV drain-
ccc.2016.12.010. PMID: 28284294. age cannula, but it is not in itself a contraindication to
ELSO Guidelines for Adult Respiratory Failure (2017). VA ECMO. Cardiogenic shock after myocardial infarc-
Extracorporeal Life Support Organization, Version 1. tion is a reasonable indication for VA ECMO. It may also
https://www.elso.org/Portals/0/ELSO%20Guidelines%20 be considered in other forms of cardiogenic shock,
For%20Adult%20Respiratory%20Failure%201_4.pdf including myocarditis, pulmonary embolism, and post-
(accessed 4 August 2017). cardiotomy. It may also be used to manage heart failure
Schmidt M, Bailey M, Sheldrake J, et al. Predicting survival with a plan to bridge to permanent ventricular assist
after extracorporeal membrane oxygenation for severe device placement or transplant.
acute respiratory failure. The Respiratory Extracorporeal
Membrane Oxygenation Survival Prediction (RESP) Answer: D
score. Am J Respir Crit Care Med. 2014;189(11):1374–
Yannopoulos D, Bartos J, Raveendran G, et al. Advanced
82. doi: https://doi.org/10.1164/rccm.201311-­2023OC.
reperfusion strategies for patients with out-­of-­hospital
PMID: 24693864.
cardiac arrest and refractory ventricular fibrillation
(ARREST): a phase 2, single centre, open-­label,
2 A 62-­year-­old man with a history of alcoholic cirrho-
randomised controlled trial. Lancet. 2020 Nov
sis (MELD 18), active alcohol abuse, mild aortic valve
12:S0140–6736(20)32338-­2. doi: https://doi.org/10.1016/
insufficiency, type II diabetes, and obesity (BMI = 35)
S0140-­6736(20)32338-­2. Epub ahead of print. PMID:
presents to the emergency department with an ST-­
33197396.
elevation MI. He is immediately taken to the cardiac
Lee SN, Jo MS, Yoo KD. Impact of age on extracorporeal
catheterization lab for percutaneous coronary inter-
membrane oxygenation survival of patients with cardiac
vention; a left anterior descending artery culprit lesion
failure. Clin Interv Aging. 2017 Aug 24;12:1347–1353.
is successfully stented. However, postprocedure, he
doi: https://doi.org/10.2147/CIA.S142994. PMID:
remains in profound shock on very high doses of intra-
28883715; PMCID: PMC5576703.
venous epinephrine, norepinephrine, and vasopressin.
Salna M, Chicotka S, Biscotti M III, et al. Morbid obesity is
Arterial blood pressure is 85/40 mm Hg. A bedside
not a contraindication to transport on extracorporeal
echocardiogram indicates significant left ventricular
support. Eur J Cardiothorac Surg. 2018;53(4):793–798.
dysfunction with an ejection fraction of 25%. The car-
doi: https://doi.org/10.1093/ejcts/ezx452. PMID:
diologist is requesting veno-­arterial (VA) ECMO given
29253111.
the patient’s shock state. Which of the following
Makdisi G, Wang IW. Extra Corporeal Membrane
patient characteristics is the strongest contraindica-
Oxygenation (ECMO) review of a lifesaving technology.
tion for providing ECMO support?
J Thorac Dis. 2015;7(7):E166–76. doi: https://doi.
A Age of 62
org/10.3978/j.issn.2072-­1439.2015.07.17. PMID:
B Morbid obesity (BMI 35)
26380745; PMCID: PMC4522501.
C Mild aortic valve insufficiency
D Alcoholic cirrhosis 3 A 45-­year-­old previously healthy man was a pedes-
E Immediately post-­MI with LV dysfunction trian struck by a motor vehicle resulting in multiple
injuries including traumatic brain injury with a suba-
This patient is a poor candidate for several reasons; how- rachnoid hemorrhage (SAH), multiple rib fractures,
ever, cirrhosis is the strongest contraindication to this pulmonary contusion, hemothorax, splenic laceration,
therapy as it portends a poor overall outcome. Chronic and a pelvic fracture. On postinjury day 5, he devel-
end-­organ dysfunction with no exit strategy (such as oped severe hypoxemic respiratory failure (PaO2:FiO2
transplant for which this patient is not a candidate given ratio of 70 on FiO2 of 100%) and was diagnosed with
his active alcohol abuse) is an absolute contraindication an MRSA pneumonia. Workup for other causes of res-
to ECMO. piratory failure or sepsis was negative, and there was
Advanced age is a relative contraindication to ECMO, no evidence of SAH progression or torso hemorrhage
with age of 65 often used as a cutoff in older literature. on his most recent imaging. His hypoxemic respiratory
However, VA ECMO in patients up to 75 years of age has failure did not improve with proning and neuromuscular
ECMO 21

blockade. What is the optimal ECMO cannulation balance is 3L negative since initiation of ECMO. His
strategy for this patient? pulmonary capillary wedge pressure is 12 mm Hg. His
A Femoral venous drainage, carotid arterial reinfusion chest x-­ray shows bilateral lower lobe infiltrates.
B Femoral venous drainage, femoral arterial reinfusion However, his upper body peripheral oxygen PaO2 is
C Femoral venous drainage, jugular venous reinfusion 40 mm Hg despite maximal ARDSnet appropriate
D Femoral venous drainage, femoral venous reinfusion ventilator settings, while his lower body PaO2
E Jugular venous drainage, right atrial reinfusion (dual remains > 200 mm Hg. What is the next best step in his
lumen cannula) management?
A Place a left ventricular microaxial percutaneous ven-
This patient has no evidence of cardiac failure, so veno-­ tricular assist device for left ventricular venting.
arterial cannulation is unnecessary. This approach increases B Increase total VA ECMO flows to improve upper
the potential for an arterial injury or thromboembolic body saturation.
event, will significantly increase the patient’s cardiac after- C Add a second-­line inopressor in addition to epinephrine.
load, and may not provide adequate oxygenation. D Place a venous reinfusion ECMO cannula and con-
The most common cannulation strategy for veno- vert the patient’s configuration to VA-­V ECMO.
venous ECMO is femoral drainage and jugular reinfu- E Perform an atrial septostomy for left ventricular
sion. A multistage, large-­bore venous drainage cannula unloading.
will adequately support the gas exchange needs for most
adult patients (4–6 L/min flow) without risking flow lim- Left ventricular venting is commonly employed in
itations or recirculation that can be a problem with the patients supported on peripheral VA ECMO when the
bilateral femoral-­femoral venovenous approach. Single native cardiac function is not robust enough to overcome
site cannulation with a dual lumen cannula facilitates the increased afterload generated by the VA ECMO cir-
early ambulation for ECMO patients; it is commonly cuit, which leads to left ventricular distention. This
used for those awaiting a lung transplant. patient shows no signs of left ventricular distention with
a normal PCWP, no signs of aortic or mitral insuffi-
Answer: C ciency, and an improving ejection fraction. Performing
LV decompression with a septostomy or mechanical
Cannon JW, Gutsche JT, Brodie D. Optimal strategies for
device is likely unnecessary in this patient.
severe acute respiratory distress syndrome. Crit Care
There is no evidence of renal or hepatic impairment
Clin. 2017;33(2):259–275. doi: https://doi.org/10.1016/j.
and cardiac function has improved, making an increase
ccc.2016.12.010. PMID: 28284294.
in cardiac output, especially to the lower body (whether
ELSO Guidelines for Adult Respiratory Failure (2017).
increased arterial flow or increased inopressor sup-
Extracorporeal Life Support Organization, Version 1.
port), unnecessary. Rather, this patient is likely suffer-
https://www.elso.org/Portals/0/ELSO%20Guidelines%20
ing from severe respiratory failure from aspiration
For%20Adult%20Respiratory%20Failure%201_4.pdf
pneumonitis rather than left-­sided heart failure and
(accessed 4 August 2017).
pulmonary edema. While his lower body oxygen deliv-
ELSO Guidelines for Cardiopulmonary Extracorporeal Life
ery is adequate, the oxygen delivery to the coronary and
Support (2017). Extracorporeal Life Support
cerebral circulation is likely not, with a PaO2 of
Organization, Version 1. https://www.elso.org/Portals/0/
40 mm Hg. Addition of a venous reinfusion limb to con-
ELSO%20Guidelines%20General%20All%20ECLS%20
vert to a hybrid VA-­V ECMO circuit will provide addi-
Version%201_4.pdf (accessed 4 August 2017).
tional oxygenation support and is the most useful next
4 A 58-­year-­old man is on day 2 of veno-­arterial ECMO step.
support after an aspiration event led to a cardiac Answer: D
arrest. He is cannulated via his left common femoral
vein for drainage and right common femoral artery for Russo JJ, Aleksova N, Pitcher I, et al. Left ventricular
reinfusion. He is on a low-­dose epinephrine infusion unloading during extracorporeal membrane oxygenation
with a blood pressure of 110/60 mm Hg and a normal in patients with cardiogenic shock. J Am Coll Cardiol.
lactate level. He has no signs of renal, hepatic, or neu- 2019;73(6):654–662. doi: https://doi.org/10.1016/j.
rologic injury. On transthoracic echocardiography, his jacc.2018.10.085. PMID: 30765031.
left ventricular ejection fraction has improved from
10% on day 1 to 30% on day 2. His left ventricular size 5 While on venovenous ECMO, which of the following
appears normal with no obvious valvular abnormali- ventilator strategies should be used to provide lung
ties. He has responded well to furosemide and his fluid protection and recovery?
22 3 Surgical Critical Care and Emergency Surgery

Figure 3.1 VA-­V ECMO circuit. Source: From


Biscotti M., Lee A,, Basner RC., et al. Hybrid
configurations via percutaneous access for
extracorporeal membrane oxygenation: a
single-­center experience. ASAIO J.
2014;60(6):635–42. with permission.

A T-­piece or tracheostomy collar results in extremely high driving pressures, especially


B High-­frequency percussive ventilation early after ECMO initiation.
C High-­frequency oscillatory ventilation Answer: E
D Volume control 8 mL/kg ideal body weight
E Pressure control with PEEP of 10 cm H20 Abrams D, Schmidt M, Pham T, et al. Mechanical
ventilation for acute respiratory distress syndrome
Lung protective ventilation should continue after ECMO during extracorporeal life support. Research and
initiation. In fact, so-­called ultra-­lung protective ventila- practice. Am J Respir Crit Care Med. 2020;201(5):514–
tion is often feasible once the majority of the patient’s gas 525. doi: https://doi.org/10.1164/rccm.201907-­1283CI.
exchange needs are provided by the ECMO circuit. The PMID: 31726013.
best current approach is likely reflected in the recently Brodie D, Bacchetta M. Extracorporeal membrane
conducted EOLIA trial in which plateau airway pressure oxygenation for ARDS in adults. N Engl J Med.
was limited to a maximum of 24 cm H2O in conjunction 2011;365(20):1905–14. doi: https://doi.org/10.1056/
with PEEP > = 10 cm H2O (corresponding to a driving NEJMct1103720. PMID: 22087681.
pressure < = 14 cm H2O), respiratory rate of 10–30 ELSO Guidelines for Cardiopulmonary Extracorporeal Life
breaths/min, and FiO2 of 0.3–0.5. This can be achieved Support (2017). Extracorporeal Life Support
with either a volume control or a pressure control mode, Organization, Version 1. https://www.elso.org/Portals/0/
but in our view, a pressure control mode is easier to apply ELSO%20Guidelines%20For%20Adult%20
in the setting of very low lung compliance. Often the Respiratory%20Failure%201_4.pdf (accessed 4 August
tidal volumes will be much lower than 4 mL/kg, espe- 2017).
cially early after ECMO initiation. Furthermore, the res-
piratory rate should be minimized to further decrease 6 A 30-­year-­old previously healthy man is placed on
ventilator-­induced lung injury. venovenous (VV) ECMO for severe COVID-­ 19
Early after ECMO initiation, patients may have sig- pneumonia. On ECMO day 5, he is intubated but
nificant air hunger and may also need moderate-­to-­ awake and interactive on minimal sedation. His
deep sedation for a period of time. As a result, morning chest x-­ray demonstrates a new right-­sided
spontaneous modes of ventilation are not employed pneumothorax. After insertion of a tube thoracos-
until the patient has shown some signs of stabilization tomy, he continues to have a large, continuous air
or even recovery. High-­frequency percussive ventila- leak on ECMO day 7. His pulmonary compliance
tion can help with mobilizing secretions, particular in remains moderate-­to-­high, with a tidal volume of
patients with inhalation injury, but this approach is not 7 mL/kg IBW on a positive end-­e xpiratory pressure
routinely used in ECMO patients. High-­ frequency (PEEP) of 5, a driving pressure of 10 cm H2O, a frac-
oscillatory ventilation has no proven benefit in this tion of inspired oxygen (FiO2) of 0.5, and a respira-
population and may actually cause harm in some cases. tory rate of 20 breaths/min. He remains on ECMO
Finally, volume control ventilation at this level typically support with a sweep gas flow rate of 6 liters/min.
ECMO 23

His peripheral arterial blood gas shows a pH of 7.36, The most common anticoagulation approach is a heparin
PaCO2 of 47, and a PaO2 of 78. What is the best bolus upon cannula insertion (50–100 units/kg) followed by
management approach for this patient’s mechanical a continuous heparin infusion (7.5-­20 units/kg/hr). Heparin
ventilation? titration has historically been performed based on activated
A Extubate to high flow nasal cannula. clotting time (ACT) measured at the bedside (target 180–
B Increase PEEP. 220 seconds); however, recent evidence suggests that either
C Convert to airway pressure release ventilation a PTT-­based approach (1.5-­2 times baseline) or an anti-­Xa
(APRV) with a PHI of 30 and PLOW of 0. approach (0.25 units/mL) may be preferable.
D Sedate, paralyze, and prone positioning. Therapeutic low molecular weight injections are not
E Increase tidal volumes. typically performed on ECMO. Argatroban, a direct
thrombin inhibitor, can be used but is generally reserved
This patient has a persistent continuous air leak, which for patients with a history of, or concern for, HITT.
can be exacerbated by continuous positive pressure ven- Withholding anticoagulation can be done as described
tilation. Ventilator strategies to aid in healing of bron- above, and some evidence suggests this may actually be
chopleural fistulae typically include lowering airway safe for the entirety of a short ECMO run. However, this
pressures and PEEP. Strategies that include increasing is not currently a standard approach. Likewise, dual anti-
PEEP, tidal volumes, or APRV can lead to higher airway platelet therapy (DAPT) alone is not a standard approach
pressures, which may preclude lung healing. In select although it may be used in patients with other indica-
cases, extubation may be a reasonable strategy, provided tions for DAPT, which is more common in patients on
the patient can be sufficiently supported without tra- veno-­arterial ECMO.
cheal intubation. Answer: A
Answer: A
ELSO Anticoagulation Guidelines (2017). Extracorporeal
Xia J, Gu S., Li M,s et al. Spontaneous breathing in patients Life Support Organization, Version 2014. https://www.
with severe acute respiratory distress syndrome elso.org/portals/0/files/elsoanticoagulationguideline8-­
receiving prolonged extracorporeal membrane 2014-­table-­contents.pdf (accessed 30 July 2021).
oxygenation. BMC Pulm Med. (2019);19:237. https://doi. Kurihara C, Walter JM, Karim A, et al. Feasibility of
org/10.1186/s12890-­019-­1016-­2 venovenous extracorporeal membrane oxygenation
without systemic anticoagulation. Ann Thorac Surg.
7 After initiating venovenous ECMO, which strategy is 2020;110(4):1209–1215. doi: https://doi.org/10.1016/j.
most likely to minimize bleeding while also prevent- athoracsur.2020.02.011. Epub 2020 Mar 12. PMID:
ing clot formation in the circuit or around the 32173339; PMCID: PMC7486253.
cannulas? Parker RI. Anticoagulation monitoring during
A Heparin bolus and infusion extracorporeal membrane oxygenation: continuing
B Low molecular weight heparin 1.5 mg/kg twice progress. Crit Care Med. 2020;48(12):1920–1921. doi:
daily https://doi.org/10.1097/CCM.0000000000004635.
C Argatroban infusion PMID: 33255117.
D Dual antiplatelet therapy Vandenbriele C, Vanassche T, Price S. Why we need safer
E Withholding systemic anticoagulation for 24 hours anticoagulant strategies for patients on short-­term
percutaneous mechanical circulatory support. Intensive
Blood exposure to the surface of the gas exchange mem- Care Med. 2020;46(4):771–774. doi: https://doi.
brane and the circuit activates the intrinsic clotting cas- org/10.1007/s00134-­019-­05897-­3. Epub 2020 Jan 23.
cade, the complement system, and platelets. This results PMID: 31974917.
in a state of both hyper-­and hypo-­coagulation. In some
cases such as a recent intracranial bleed or solid organ 8 Since the inception of ECMO technology in the 1970s,
injury, patients on venovenous ECMO may have antico- the rates of bleeding and thrombotic complications
agulation withheld. However, in most cases, low-­dose have decreased significantly, though they remain a
anticoagulation is used to preserve the gas exchange significant cause of morbidity and mortality. Which
membrane’s efficiency, increase the circuit longevity, factor is likely the most significant contributor to the
and mitigate the risk of thromboembolic complications. observed decrease in bleeding and thrombotic compli-
Patients on veno-­arterial ECMO are generally main- cations over the past several decades?
tained on higher doses of anticoagulation given the A Novel anticoagulants including direct thrombin
more significant implications of an arterial thromboem- inhibitors
bolic event. B Changes in ECMO device technologies
24 3 Surgical Critical Care and Emergency Surgery

C The invention and use of thromboelastography It is safe and feasible to provide RRT via either separate vas-
D More accurate assays for activated clotting time cular access or direct integration into the ECMO circuit,
and activated partial thromboplastin time depending on patient-­specific circumstances. However, the
E Discovery of modern-­day antiplatelet therapy polymethylpentene oxygenator will provide gas exchange
but will not function as a hemofilter to provide RRT.
The use of novel anticoagulants and antiplatelet thera- Answer: E
pies in ECMO has been described but has not been stud-
ied sufficiently to make any recommendations for or Ostermann M, Connor M Jr, Kashani K. Continuous renal
against their use. replacement therapy during extracorporeal membrane
Use of TEG and ACT monitors, as well as protocols tar- oxygenation: why, when and how? Curr Opin Crit Care.
geting low or high PTT goals, is often implemented; how- 2018;24(6):493–503. doi: https://doi.org/10.1097/
ever, current evidence is insufficient to recommend one MCC.0000000000000559. PMID: 30325343.
specific approach over the others. The improvements in Gorga SM, Sahay RD, Askenazi DJ,et al. Fluid overload and
ECMO circuit technology and heparin-­coated cannulas fluid removal in pediatric patients on extracorporeal
have likely led to a decrease in total dose and duration of membrane oxygenation requiring continuous renal
anticoagulation required and an improvement in circuit-­ replacement therapy: a multicenter retrospective cohort
related hemorrhagic or thrombotic complications. study. Pediatr Nephrol. 2020;35(5):871–882. doi: https://
Answer: B doi.org/10.1007/s00467-­019-­04468-­4. Epub 2020 Jan 17.
PMID: 31953749; PMCID: PMC7517652.
Sklar MC, Sy E, Lequier L, et al. Anticoagulation practices Dado DN, Ainsworth CR, Thomas SB, et al. Outcomes
during venovenous extracorporeal membrane among patients treated with renal replacement therapy
oxygenation for respiratory failure. A systematic review. during extracorporeal membrane oxygenation: a
Ann Am Thorac Soc. 2016;13(12):2242–2250. doi: single-­center retrospective study. Blood Purif.
https://doi.org/10.1513/AnnalsATS.201605-­364SR. 2020;49(3):341–347. doi: https://doi.
PMID: 27690525. org/10.1159/000504287. Epub 2019 Dec 19. PMID:
31865351; PMCID: PMC7212702.
9 Acute kidney injury (AKI) is a common problem in
patients requiring ECMO therapy. As such, the use of 10 A 40-­year-­old man is placed on venovenous (VV)
renal replacement therapy (RRT) is necessary in ECMO via a 25 Fr right femoral vein drainage can-
40–60% of cases. Which of the following statements nula and a 17 Fr right internal jugular vein reinfu-
regarding use of RRT and ECMO is most accurate? sion cannula for refractory ARDS secondary to
A RRT access should never be provided via an in-­line aspiration pneumonitis. He is 6’ 2” tall and weighs
approach with ECMO circuits. It should always be 240 lbs (BMI 30.8 kg/m2, BSA 2.35 m2). His initial
provided via separate vascular access. circuit flow is 5.0 L/min at an RPM of 4000 and
B Fluid overload is an uncommon problem in the drainage pressure of −120 cm H2O; the ECMO spe-
pediatric ECMO population and has no significant cialist is unable to flow > 5.0 L/min because of exces-
effect on morbidity and mortality. sively high drainage pressures (chatter) in the line.
C Uremia and electrolyte derangements are the most Over the next 48 hours, his SpO2 remains at 70% on
common indications for RRT initiation in both chil- maximal ventilator settings with a hemoglobin of
dren and adults on ECMO. 14 g/dL; no signs of untreated sepsis, infection, or
D The polymethylpentene oxygenator used in ECMO shock; normal biventricular function on echocardio-
circuits can also be used as a hemofilter to deliver gram, and a persistently elevated lactate. His circuit
RRT in patients with concomitant AKI. flows remain the same and the oxygenator health is
E Negative fluid balance on RRT is independently excellent. What is the next most appropriate step?
associated with improved outcomes for both the A Consider adding an additional arterial reinfusion
adult and pediatric ECMO population. limb to provide increased ECMO support.
B Consider adding a 21Fr venous reinfusion limb to
The most common indication for RRT in both adult and provide increased ECMO support.
pediatric ECMO patients is fluid overload. Specifically, C Transfuse the patient to a supranormal hemo-
in the pediatric population, fluid overload is associated globin to improve oxygen delivery.
with increased mortality and longer duration of ECMO D Begin aggressive intravenous fluid resuscitation to
support. Further, several studies have associated a net improve circuit venous drainage.
negative fluid balance while on RRT with improved E Consider adding an additional drainage cannula
patient outcomes. to increase overall ECMO flows.
ECMO 25

Inadequate ECMO flows is a common problem, and 0.5 L/min with the same device RPMs. What is the
because of fluid dynamics, venous drainage (access) most likely explanation for this finding?
insufficiency is typically the limiting factor rather than A The oxygenator efficiency has decreased.
reinfusion cannula size. Venous drainage pressures B There is inadequate oxygen delivery to the tissue
more negative than −100 mm Hg are typically associated resulting in tissue hypoxia.
with “chatter” in the lines and, therefore, flow limita- C The patient now has severe ARDS.
tions. Conversely, flow is often not limited by reinfusion D The patient’s heart and left ventricular ejection
pressures until the reinfusion line pressure is > 300– fraction are beginning to recover.
400 mm Hg. In patients with drainage insufficiency, the E The right radial blood gas is likely venous.
addition of a venous or arterial reinfusion limb will not
increase ECMO flows and will not provide any addi- As the cardiac function improves in patients on periph-
tional benefit. eral VA ECMO, the native cardiac output will compete
Some ECMO physicians advocate for transfusions to with retrograde aortic ECMO flow, thereby “pushing”
normal hemoglobin levels, instead of using typical ICU left ventricular blood further across the aortic arch. This
transfusion practices with a transfusion threshold of 7 or phenomenon of moving the mixing point more distally
8 g/dL. However, supplementing the patient’s already into the aortic arch demonstrates the “Harlequin syn-
normal hemoglobin (14 g/dL) is unlikely to add addi- drome” that is often seen with femoral-­femoral veno-­
tional benefit. Additionally, patients with severe ARDS arterial ECMO. A sample of arterial blood from a right
typically benefit from volume removal rather than vol- radial arterial line may demonstrate a more “normal”
ume expansion. While fluid boluses may temporarily PaO2 rather than the supranormal PaO2 that is indicative
improve flows by improving venous drainage, this is not of ECMO circuit blood, and this is often a sign that the
an effective long-­term solution. cardiac function is beginning to recover. Because most
In patients with a large body size, they may require modern ECMO circuits utilize an afterload-­ sensitive
higher than typical ECMO flows, and addition of an centrifugal pump, total VA ECMO flows will often
extra drainage cannula via the contralateral femoral vein decrease as cardiac function improves and the circuit
may improve total circuit flow capacity, which will miti- afterload increases.
gate the hypoxemia and resultant tissue hypoxia. A high post-­oxygenator PaO2 suggests adequate and
unchanged oxygenator function, and so long as end-­
Answer: E organ perfusion remains normal, it is unlikely that tissue
hypoxia is occurring.
Dado DN, Ainsworth CR, Thomas SB, et al. Outcomes
among patients treated with renal replacement therapy
A PaO2:FiO2 ratio > 300 does not meet clinical criteria
during extracorporeal membrane oxygenation: a
for severe ARDS, and a PaO2 > 100 mm Hg is unlikely to
single-­center retrospective study. Blood Purif.
be from a venous blood sample.
2020;49(3):341–347. doi: https://doi.
Answer: D
org/10.1159/000504287. Epub 2019 Dec 19. PMID:
31865351; PMCID: PMC7212702. Eckman PM, Katz JN, El Banayosy A, et al. Veno-­Arterial
extracorporeal membrane oxygenation for cardiogenic
11 A 55-­year-­old man is emergently placed on femoral-­ shock: an introduction for the busy clinician.
femoral veno-­ arterial (VA) ECMO for a cardiac Circulation. 2019;140(24):2019–2037. doi: https://doi.
arrest caused by an acute MI. The culprit coronary org/10.1161/CIRCULATIONAHA.119.034512. Epub
lesion was stented in the cardiac catheterization lab, 2019 Dec 9. PMID: 31815538.
and he was taken to the ICU to recover. On hospital
day 1, his post-­oxygenator PaO2 is 400 mm Hg and 12 Which of the following sites of hemorrhage is most
radial arterial PaO2 is also 400 mm Hg. Transthoracic common during ECMO support?
echocardiogram demonstrates a left ventricular ejec- A Intracranial
tion fraction of 10%. He remains intubated with a B Cannula site
positive end-­expiratory pressure (PEEP) of 5 cm H2O C Solid organ
and a fraction of inspired oxygen (FiO2) of 40%. On D Gastrointestinal
hospital day 3, with the same ventilator settings, his E Pulmonary
right radial arterial line demonstrates a PaO2 of
150 mm Hg and his post-­oxygenator blood gas PaO2 Bleeding complications occur in approximately 24% of
remains at 400 mm Hg. His lactate levels remain ECMO patients. The ELSO registry records these com-
normal. The total ECMO flows have decreased by plications. Participation in this registry is voluntary;
26 3 Surgical Critical Care and Emergency Surgery

https://doi.org/10.1001/jama.2019.9302. PMID:
31408142.

13 General and trauma surgeons are often called upon


to consult on ECMO patients. Which statement
about surgical procedures for ECMO patients is
most accurate?
A Intra-­abdominal operations should not be per-
formed on ECMO patients, as intraoperative mor-
tality typically exceeds 50%.
Aorta
LA
B Tracheostomy can be performed safely and effec-
RA
LV
tively on ECMO patients.
RV
C The use of antifibrinolytic agents (such as amino-
VA-ECMO
caproic acid or tranexamic acid) for mitigating
bleeding in ECMO patients has not been described.
D Interruption of anticoagulation for any period of time
will result in immediate catastrophic circuit failure.
E Caesarean section is absolutely contraindicated
during ECMO support.
Arterial
sampling Noncardiac surgical procedures on ECMO patients are
common and are required in as many as 50% of patients.
Most studies have not demonstrated increased mortality
among patients who underwent surgical procedures. Use of
ECMO in pregnancy is also well described with Caesarean
sections successfully performed in patients on ECMO.
One of the most common procedures performed on
ECMO patients is tracheostomy, and many reviews have
Figure 3.2 Harlequin syndrome. Source: From Eckman PM, Katz demonstrated safety and efficacy of tracheostomy on
JN, El Banayosy A, et al. Veno-­arterial extracorporeal membrane ECMO patients. In fact, tracheostomy may decrease
oxygenation for cardiogenic shock: an introduction for the busy
clinician. Circulation. 2019;140(24):2019–2037, with permission. sedation requirements and decrease time on ECMO.
Bleeding is still a risk; however, it is our approach to use
electrocautery on the skin and soft tissue down to the
however, in some instances, the exposure may not be trachea, turning the ventilator FiO2 < 60% and then pro-
100% of patients resulting in an under-­representation of ceeding with a percutaneous tracheostomy technique.
certain types of bleeding complications (e.g. the percent- Additionally, in many centers, antifibrinolytic medica-
age of surgical site bleeding may be artificially lowered by tions are used prophylactically during surgical proce-
including nonsurgical patients in the denominator). dures or to treat bleeding complications in ECMO
However, it appears that cannula site hemorrhage is the patients, and it is unlikely that short-­term interruption of
most common bleeding complication (8%) followed by anticoagulation infusions will cause circuit failure.
surgical site bleeding (7%), gastrointestinal bleeding
Answer: B
(6%), pulmonary hemorrhage (4%), and central nervous
system bleeding (3%). Juthani BK, Macfarlan J, Wu J, et al. Incidence of general
Strategies for managing cannula site bleeding include surgical procedures in adult patients on extracorporeal
prevention by under-­sizing the insertion site incision for membrane oxygenation. J Intensive Care Soc. 2019
cannula insertion, application of topical hemostatics May;20(2):155–160. doi: https://doi.
around the cannula site, placement of purse string org/10.1177/1751143718801705. Epub 2018 Oct 2.
sutures around the insertion site, or lowering the antico- PMID: 31037108; PMCID: PMC6475990.
agulation target. Salna M, Tipograf Y, Liou P, et al. Tracheostomy is safe
Answer: B during extracorporeal membrane oxygenation support.
ASAIO J. 2020;66(6):652–656. doi: https://doi.
Brodie D, Slutsky AS, Combes A. Extracorporeal life org/10.1097/MAT.0000000000001059. PMID: 31425269.
support for adults with respiratory failure and related Buckley LF, Reardon DP, Camp PC, et al. Aminocaproic
indications: a review. JAMA. 2019;322(6):557–568. doi: acid for the management of bleeding in patients on
ECMO 27

extracorporeal membrane oxygenation: four adult case 15 A 40-­ year-­


old man with severe influenza-­ induced
reports and a review of the literature. Heart Lung. ARDS is placed on venovenous ECMO via left and
2016;45(3):232–6. doi: https://doi.org/10.1016/j. right common femoral veins. He requires ECMO for
hrtlng.2016.01.011. Epub 2016 Feb 20. PMID: 26907195. 12 days, and throughout his course is maintained on a
Agerstrand C, Abrams D, Biscotti M, et al. Extracorporeal continuous heparin infusion with an average acti-
membrane oxygenation for cardiopulmonary failure during vated partial thromboplastin time (aPTT) of 60 sec-
pregnancy and postpartum. Ann Thorac Surg. onds. He is successfully decannulated after
2016;102(3):774–779. doi: https://doi.org/10.1016/j. improvement in lung function, and then maintained
athoracsur.2016.03.005. Epub 2016 May 4. PMID: 27154158. on a continuous heparin infusion for 48 hours after
decannulation. Two days after decannulation the
14 A 45-­year-­old woman is placed on femoral-­femoral patient has sudden-­ onset tachycardia, hypoxemia,
veno-­arterial ECMO via a 25 Fr venous cannula in the and hypotension. There is no change in the physical
left femoral vein and a 17 Fr arterial cannula in the exam, respiratory mechanics, or chest x-­ray. What is
right femoral artery. Six hours after cannulation, the the next most appropriate step?
patient remains on moderate-­dose inopressors and is A Discontinue heparin as the patient may be bleed-
well supported with an ECMO flow of 3.5 L/min. The ing from the cannulation sites.
bedside nurse notices mottling of the right foot. What is B Perform a transthoracic echocardiogram and
the most likely etiology and reasonable next step? consider CT chest to evaluate for a pulmonary
A The patient has a right femoral DVT and is devel- embolism.
oping phlegmasia cerulea dolens. She requires a C Place bilateral chest tubes.
venous thrombolysis procedure. D Make the patient DNR after a family meeting.
B The patient has decreased cardiac output and E Perform an emergent bronchoscopy.
requires an increase in inopressors.
C The arterial reinfusion cannula has thrombosed Venous thromboembolism is a very common compli-
and must be replaced immediately. cation after venovenous ECMO. It occurs in 30–50% of
D The arterial cannula is causing distal limb patients after decannulation, even despite appropriate
ischemia. A distal perfusion catheter should be anticoagulation. Pulmonary embolism should be high
placed emergently. on every clinicians’ differential diagnosis and should
E The patient has an ischemic foot and a below-­the-­ be ruled out and must be suspected and potentially
knee amputation should be performed. treated.
Other causes of respiratory failure to include pneumo-
The incidence of distal limb ischemia is 10–70% in thorax, worsening lung function, or mucous plugging are
peripheral VA ECMO patients. It is associated with an also common in patients recovering from severe lung
increased risk of morbidity and mortality. It must be rec- disease and must be ruled out as well with physical exam,
ognized and treated urgently by placement of a distal observation of respiratory mechanics, and chest x-­ray.
perfusion catheter/cannula. If not recognized promptly, Goals of care discussions are always valuable in the
amputation may be required; however, reperfusion to management of critically ill patients; however, in the
the ischemic limb should be attempted prior to any con- young, recovering patient, a family meeting to address
sideration for amputation. DNR status is probably premature.
It is possible to have a DVT leading to phlegmasia in
ECMO patients; however, it is more likely to occur in the Answer: B
leg with the venous cannula and unlikely immediately
Trudzinski FC, Minko P, Rapp D, et al. Runtime and aPTT
after cannulation.
predict venous thrombosis and thromboembolism in
It is unlikely that decreases in cardiac output or insuf-
patients on extracorporeal membrane oxygenation: a
ficient/absent ECMO flows would result in localized
retrospective analysis. Ann Intensive Care. 2016;6(1):66.
ischemia.
doi: https://doi.org/10.1186/s13613-­016-­0172-­2. Epub
Answer: D 2016 Jul 19. PMID: 27432243; PMCID: PMC4949188.

Bonicolini E, Martucci G, Simons J, et al. Limb ischemia in 16 Which of the following is the strongest clinical indi-
peripheral veno-­arterial extracorporeal membrane cation to discontinue ECMO support?
oxygenation: a narrative review of incidence, prevention, A A patient is intubated for 16 days and requires a
monitoring, and treatment. Crit Care. 2019;23(1):266. tracheostomy procedure.
doi: https://doi.org/10.1186/s13054-­019-­2541-­3. PMID: B Arterial blood gas demonstrates a pH of 7.36 and a
31362770; PMCID: PMC6668078. PaCO2 of 55 mm Hg on a sweep gas flow of 4 L/min.
28 3 Surgical Critical Care and Emergency Surgery

C The patient has been on ECMO for 2 weeks. in ICU care and device technology; while the survival-­
D The patient is oozing blood from a left chest tube to-­discharge for cardiac failure ECMO is approximately
site and right femoral cannulation site. 53%. Though data is limited and premature, the
E The patient has an SpO2 of 96% and arterial survival-­to-­
discharge of ECMO patients with
PaCO2 of 40 mm Hg on 0 L/min of VV ECMO COVID-­19 is 54%. While these survival rates are all
sweep gas flow and low ventilator settings. encompassing and have been gathered over several
decades, there are several prediction tools to attempt to
As a rule of thumb, when extracorporeal support pro- elucidate anticipated survival for the individual patient;
vides less than 30% of native cardiac or lung function, a one of which is the RESP score. This model uses data
trial off ECMO is indicated. If SpO2 > 95%, and arterial points including age, duration of mechanical ventila-
PaCO2 is < 50mm Hg for > 60 min off of sweep flow, tion, immunocompromised status, among several other
decannulation from VV ECMO is reasonable. Patients patient-­specific data points.
with an elevated PaCO2 despite moderate sweep gas flow Except in cases of severe device-­related complica-
are likely not ready for a trial off ECMO. tions, patients decannulated from ECMO should be
The need for a surgical procedure alone is not an indi- adequately and safely maintained on an amount of sup-
cation for decannulation. In some cases, ECMO is indi- port that allows for expedient recovery, re-­conditioning,
cated to provide additional support to patients and physical therapy. If it is anticipated that a patient
undergoing high-­risk surgical procedures (such as com- requires neuromuscular blockade, increased sedation,
plex airway or tracheal reconstructions or resections of and increased ventilator settings, then they should not
anterior mediastinal masses). Additionally, prolonged be decannulated. The in-­hospital mortality after ECMO
duration of ECMO support should not be an isolated decannulation is approximately 10%. In select cases,
reason for decannulation. patients who were decannulated from ECMO may
A small amount of oozing from surgical sites is not require a second ECMO run, and this is within reason.
uncommon in ECMO patients. Premature decannula- Approximately 30–50% of patients decannulated from
tion may be considered only in rare cases of uncontrol- VV ECMO will suffer from a DVT, and screening for
lable bleeding. DVT is typically performed 48–72 hours post-­ECMO
decannulation.
Answer: E
Answer: C
ELSO Guidelines for Cardiopulmonary Extracorporeal Life
Support (2017). Extracorporeal Life Support ELSO Guidelines for Cardiopulmonary Extracorporeal Life
Organization, Version 1. Ann Arbor, MI, USA. www. Support (2017). Extracorporeal Life Support
elso.org (accessed 4 August 2017). Organization, Version 1. Ann Arbor, MI, USA. www.
elso.org (accessed 4 August 2017).
Trudzinski FC, Minko P, Rapp D, et al. Runtime and aPTT
17 A 35-­year-­old woman suffering from COVID-­19 is
predict venous thrombosis and thromboembolism in
decannulated from venovenous (VV) ECMO after
patients on extracorporeal membrane oxygenation: a
12 days. She remains on the ventilator and in the
retrospective analysis. Ann Intensive Care. 2016;6(1):66.
ECMO ICU. The family is asking what they can
doi: https://doi.org/10.1186/s13613-­016-­0172-­2. Epub
expect for her post-­ECMO course. Which statement
2016 Jul 19. PMID: 27432243; PMCID: PMC4949188.
is most accurate?
Schmidt M, Bailey M, Sheldrake J, et al. Predicting survival
A Approximately 40% of patients who are decannu-
after extracorporeal membrane oxygenation for severe
lated from ECMO will ultimately die in the hospital.
acute respiratory failure. The Respiratory Extracorporeal
B She will require more sedation and higher ventila-
Membrane Oxygenation Survival Prediction (RESP)
tor settings in the coming days.
score. Am J Respir Crit Care Med.
C Approximately 40% of patients will suffer from a
2014;189(11):1374–82.
DVT post-­ECMO decannulation.
D Prior ECMO cannulation is a contraindication to
future ECMO cannulation. 18 A 6-­year-­old previously healthy girl is admitted to the
E Physical therapy is contraindicated in the week PICU after being involved in a house fire resulting in
post-­ECMO for fear of cannula site bleeding. acute respiratory distress with severe hypoxemic
respiratory failure. Which of the following would indi-
The survival-­to-­discharge for all-­comers in respiratory cate a need for venovenous ECMO in this patient?
failure ECMO is approximately 60%, though this rate A PaO2/FiO2 > 100–150
continues to improve year-­to-­year with improvements B Oxygenation index (OI) > 40
ECMO 29

C Mean airway pressure > 15 cmH2O on high-­ and hypoxemic (PaO2 = 42 mm Hg). He is transitioned
frequency oscillatory ventilation from a conventional ventilator to the high-­frequency
D Mean airway pressure > 15 cmH2O on conven- oscillatory ventilator. However, this results in minimal
tional ventilation improvement in gas exchange with worsening meta-
E Carboxyhemoglobin level of 10% bolic acidosis and a rising lactate on escalating vaso-
pressor support. What is the most appropriate ECMO
When evaluating a patient’s candidacy for extracorpor- cannulation strategy for this neonate?
eal support, the provider must consider the underlying A Right femoral venous drainage, left femoral artery
pathology, the adequacy of gas exchange given the cur- reinfusion
rent mechanical ventilatory requirement, and the suc- B Right femoral venous drainage, umbilical vein
cess/failure of adjunctive rescue therapies. Although reinfusion
significant variability in institutional protocols exists, C Right femoral venous drainage, right carotid
salvage therapies for children on a conventional ventila- artery reinfusion
tor with mean airway pressure (MAP) > 20–25 cm H2O D Right internal jugular vein drainage, right carotid
includes use of high-­ frequency oscillatory ventilation artery reinfusion
(HFOV), nitric oxide, and prone positioning. E Right internal jugular vein drainage and reinfu-
MAP < 30 cm H2O are tolerable while on HFOV. The sion with double-­lumen bicaval catheter
PaO2/FiO2 is the ratio of arterial oxygen partial pressure
to fractional inspired oxygen and is a clinical indicator of Extracorporeal support for VA ECMO requires veno-­
hypoxemia (normal PaO2/FiO2 > 300). An alternative arterial access. In most situations, specific cannula selec-
measure of oxygenation is the oxygenation index (OI), tion occurs after surgical cut down with direct visual
which is calculated as the reciprocal of the PaO2/FiO2 interrogation of the vessels of interest. Although some
times 100 times the mean airway pressure: centers have started to implement percutaneous cannula-
tion using Seldinger technique, this is best suited for fem-
1 oral access that is inappropriate for children < 15 kg
OI 100 MAP because of the size of the femoral vessels (answers A and C
PaO2
are therefore incorrect). Venous drainage via cannulation
FiO2 of the right internal jugular vein and reinfusion via the
FiO2 right common carotid artery is the standard approach for
OI 100 MAP
PaO2 children < 15 kg (answer D). Right femoral venous drain-
age and umbilical vein reinfusion is not described as a
Severe respiratory failure as evidence by a sustained mode of ECMO support. An umbilical vein catheter
PaO2/FiO2 < 60–80 or OI > 40 predict high mortality and (UVC) can be used for infusions on a short-­term basis but
indicate a need for lung rescue with ECMO. For example, if has not been described for ECMO support (answer B).
the patient’s PaO2 were 60 mm Hg on an FiO2 of 1 and MAP Finally, right internal jugular venous drainage and reinfu-
of 30 cm H20, the OI would be 50, which is a strong indica- sion with a double-­lumen bicaval catheter can be used for
tion for ECMO initiation in the pediatric population. VV ECMO but is inappropriate to support the child neces-
sitating both pulmonary and cardiac support (answer E).
Answer: B
Answer: D
Maratta C, Potera RM, van Leeuwen G, et al.
Extracorporeal Life Support Organization (ELSO): 2020 Johnson K, Jarboe MD, Mychaliska GB, et al. Is there a best
pediatric respiratory ELSO guideline. ASAIO J Am Soc approach for extracorporeal life support cannulation: a
Artif Intern Organs 1992. 2020;66(9):975–979. doi: review of the extracorporeal life support organization. J
https://doi.org/10.1097/MAT.0000000000001223 Pediatr Surg. 2018;53(7):1301–1304. doi:https://doi.
Zabrocki LA, Brogan TV, Statler KD, et al. Extracorporeal org/10.1016/j.jpedsurg.2018.01.015
membrane oxygenation for pediatric respiratory failure: Wild KT, Rintoul N, Kattan J, et al. Extracorporeal Life
survival and predictors of mortality. Crit Care Med. Support Organization (ELSO): guidelines for neonatal
2011;39(2):364–370. doi: https://doi.org/10.1097/ respiratory failure. ASAIO J Am Soc Artif Intern Organs
CCM.0b013e3181fb7b35 1992. 2020;66(5):463–470. doi:https://doi.org/10.1097/
MAT.0000000000001153
19 A newborn male infant with a fetal diagnosis of con-
genital diaphragmatic hernia (CDH) is admitted to 20 Monitoring of which of the following anticoagulation
the neonatal intensive care unit immediately after assays is independently associated with prolonged
delivery where he is found to be acidotic (pH = 7.1) circuit life in children on ECMO support?
Another random document with
no related content on Scribd:
Montalvo, Alfonso Diaz de, 146
Montpensier, Count of, 361, 362
Moriscos, the, 284
Mudejares, the, 15, 196, 271, et seq.
Muladies, the, 170
Muley Abul Hacen, 162, 163, 164, 167, 169, et seq., 198, 202, 203
N
Naples, 349, 350, 354, 356, 357, 361, 362, 364, 365, 366
Naples, Joanna II. of, 25
Navarre, 37, 40, 339, 388
O
Olito, Treaty of, 47
Olmedo, battle of, 64
Ordenanzas Reales, 146
Ovando, Nicholas de, 316
P
Painting, Castilian, 418–419
Palencia, Alonso de, 411
Paredes, Count of, 105, 153
Passage of Arms, 33
Paul II. Pope, 79, 85
Perez, Fra Juan, 299, 300
Philip, Archduke of Austria, 341, 375, 379, 389, 390
Pinzon, The Brothers, 303
Pius II., Bull of Pope, 78, 81
Plasencia, Count of (Duke of Arévalo), 93, 96, 98, 110
Polyglot-Bible, 403, 406
Printing, introduction of, 401
Pulgar, Hernando de (“He of the Exploits”), 225, 226;
(Author), 412, 413
Q
Quintanilla, Alonso de, 295
R
Ramirez, Francisco, 192, 283
Rapallo, sack of, 356
Rojas, Fernando de, 416
Ronda, 201, 202, 281
Royal Council, the, 142, 143
Roussillon and Cerdagne, 47, 75, 82, 111, 186, 346, 351, 352, 379
S
Salamanca, Treaty of, 389
Sanbenito, 256
Santa Cruz, College of, 404
Santa Fé, 226, 227
Santa Hermandad, La, 123, et seq.; 131, 132
Santiago, Mastership of. See Military Orders
Segovia, 19, 65, 112
Sforza, Gian Galeazzo, 348, 349, 357
Silva, Alonso de, 359
Sixtus IV., Pope, 85, 117, 118, 237, 254
Suprema, La, 259
T
Talavera, Fra Fernando de, 119, 151, 241, 272, 277, 278, 305, 323, 326
Tendilla, Count of, 272, 276, 278, 305, 404
Toledo, Cortes of, 141, et seq.
Tordesillas, Treaty of, 307
Toro, battle of, 108;
citadel of, 102
Toros de Guisandos, 67
Torquemada, Thomas de, 258, 261, 266
V
Velez-Malaga, 161, 204, 208, 209
Venegas, Cacim, 171, 180
Venice, League of, 360
Vespucci, Amerigo, 317
Villahermosa, Alfonso, Duke of, 103, 125, 175
Villena, Marquis of (Juan Pacheco), 28, 29, 30, 36, 53, 56, 61, 62, 63,
67, 70, 80, 82, 84, 86, 87;
(the younger), 87, 93, 94, 96, 98, 100, 105, 111
Y
Yañez, Alvar, 135
Z
“Zagal, Abdallah, El,” 173, 181, 201, 203, 209, 215, 216, 220, 221
Zahara, 163, 164, 200
Zamora, 98, 107
Zoraya, 171, 172
Zurita, 413
A Selection from the Catalogue of
G. P. PUTNAM’S SONS

Complete Catalogue sent on application

Old
Court Life in Spain

By Frances M. Elliot
Author of “Old Court Life in France,” etc.

2 vols. With 8 Photogravures and 48 Other Illustrations.


$5.00
The author presents a picturesque record of the romantic
early days of courtly Spain, concluding her narrative with the
reign of Ferdinand and Isabella and the new era of things
ushered in by them. There pass before the reader’s gaze the
martial figures of the Gothic kings, those barbarians of the
North who ruled Spain for two centuries and a half, forgetting,
under the spell of its sunny clime, the rugged gods that received
the worship of their ancestors. With the downfall of Roderick,
the Moors sweep into the country and henceforth the history is
that of a divided land, where the Cross is elevated against the
Crescent, where clash of sword and scimitar frequently
resounds, and Saracen warriors vie in gallantry and skill with
those who do battle for the Faith.
Old
Court Life in France

By Frances Elliot
Author of “Old Court Life in Spain,” etc.

Two Volumes. Octavo. With 60 Photogravure and Other


Illustrations. Net $5.00. Carriage 50 cents
“Probably the most readable and successful work of the kind
ever attempted. It has been through many editions, and its
popularity remains unimpaired. The work is charming in
manner and carries with it the impress of accuracy and careful
investigation.
“Mrs. Elliot’s is an anecdotal history of the French Court from
Francis I to Louis XIV. She has conveyed a vivid idea of the
personalities touched upon, and her book contains a great deal
of genuine vitality.”—Detroit Free Press.

The Crises
in the History of
The Papacy
By

Joseph McCabe
Author of “Peter Abélard,” “A Candid History of the Jesuits,”
etc.
This volume comprises a study of twenty of the most famous
of the Popes whose careers and whose influence was most
important in the development of the Church as it was in the
history of the world.

Alfred the Great


Maker of England

848–899 A. D.

By

Beatrice A. Lees
Of Somerville College, Oxford

8vo. $2.50
Few royal lives have been richer in fulfillment and more
decisive in their influence upon the course of history than that
of Alfred. Under his leadership, Wessex became the nucleus of
England’s expansion, the Danish invader was checked, and the
first beginnings of a royal navy were made. His is the most
important name in English history prior to the coming of the
Normans.
New York G. P. Putnam’s Sons London
TRANSCRIBER’S NOTES
1. P. 18, corrected "An occasion of" to "On an occasion of".
2. Silently corrected obvious typographical errors and
variations in spelling.
3. Retained archaic, non-standard, and uncertain spellings
as printed.
*** END OF THE PROJECT GUTENBERG EBOOK ISABEL OF
CASTILE AND THE MAKING OF THE SPANISH NATION, 1451-
1504 ***

Updated editions will replace the previous one—the old editions


will be renamed.

Creating the works from print editions not protected by U.S.


copyright law means that no one owns a United States copyright
in these works, so the Foundation (and you!) can copy and
distribute it in the United States without permission and without
paying copyright royalties. Special rules, set forth in the General
Terms of Use part of this license, apply to copying and
distributing Project Gutenberg™ electronic works to protect the
PROJECT GUTENBERG™ concept and trademark. Project
Gutenberg is a registered trademark, and may not be used if
you charge for an eBook, except by following the terms of the
trademark license, including paying royalties for use of the
Project Gutenberg trademark. If you do not charge anything for
copies of this eBook, complying with the trademark license is
very easy. You may use this eBook for nearly any purpose such
as creation of derivative works, reports, performances and
research. Project Gutenberg eBooks may be modified and
printed and given away—you may do practically ANYTHING in
the United States with eBooks not protected by U.S. copyright
law. Redistribution is subject to the trademark license, especially
commercial redistribution.

START: FULL LICENSE


THE FULL PROJECT GUTENBERG LICENSE
PLEASE READ THIS BEFORE YOU DISTRIBUTE OR USE THIS WORK

To protect the Project Gutenberg™ mission of promoting the


free distribution of electronic works, by using or distributing this
work (or any other work associated in any way with the phrase
“Project Gutenberg”), you agree to comply with all the terms of
the Full Project Gutenberg™ License available with this file or
online at www.gutenberg.org/license.

Section 1. General Terms of Use and


Redistributing Project Gutenberg™
electronic works
1.A. By reading or using any part of this Project Gutenberg™
electronic work, you indicate that you have read, understand,
agree to and accept all the terms of this license and intellectual
property (trademark/copyright) agreement. If you do not agree to
abide by all the terms of this agreement, you must cease using
and return or destroy all copies of Project Gutenberg™
electronic works in your possession. If you paid a fee for
obtaining a copy of or access to a Project Gutenberg™
electronic work and you do not agree to be bound by the terms
of this agreement, you may obtain a refund from the person or
entity to whom you paid the fee as set forth in paragraph 1.E.8.

1.B. “Project Gutenberg” is a registered trademark. It may only


be used on or associated in any way with an electronic work by
people who agree to be bound by the terms of this agreement.
There are a few things that you can do with most Project
Gutenberg™ electronic works even without complying with the
full terms of this agreement. See paragraph 1.C below. There
are a lot of things you can do with Project Gutenberg™
electronic works if you follow the terms of this agreement and
help preserve free future access to Project Gutenberg™
electronic works. See paragraph 1.E below.
1.C. The Project Gutenberg Literary Archive Foundation (“the
Foundation” or PGLAF), owns a compilation copyright in the
collection of Project Gutenberg™ electronic works. Nearly all the
individual works in the collection are in the public domain in the
United States. If an individual work is unprotected by copyright
law in the United States and you are located in the United
States, we do not claim a right to prevent you from copying,
distributing, performing, displaying or creating derivative works
based on the work as long as all references to Project
Gutenberg are removed. Of course, we hope that you will
support the Project Gutenberg™ mission of promoting free
access to electronic works by freely sharing Project
Gutenberg™ works in compliance with the terms of this
agreement for keeping the Project Gutenberg™ name
associated with the work. You can easily comply with the terms
of this agreement by keeping this work in the same format with
its attached full Project Gutenberg™ License when you share it
without charge with others.

1.D. The copyright laws of the place where you are located also
govern what you can do with this work. Copyright laws in most
countries are in a constant state of change. If you are outside
the United States, check the laws of your country in addition to
the terms of this agreement before downloading, copying,
displaying, performing, distributing or creating derivative works
based on this work or any other Project Gutenberg™ work. The
Foundation makes no representations concerning the copyright
status of any work in any country other than the United States.

1.E. Unless you have removed all references to Project


Gutenberg:

1.E.1. The following sentence, with active links to, or other


immediate access to, the full Project Gutenberg™ License must
appear prominently whenever any copy of a Project
Gutenberg™ work (any work on which the phrase “Project
Gutenberg” appears, or with which the phrase “Project
Gutenberg” is associated) is accessed, displayed, performed,
viewed, copied or distributed:

This eBook is for the use of anyone anywhere in the United


States and most other parts of the world at no cost and with
almost no restrictions whatsoever. You may copy it, give it
away or re-use it under the terms of the Project Gutenberg
License included with this eBook or online at
www.gutenberg.org. If you are not located in the United
States, you will have to check the laws of the country where
you are located before using this eBook.

1.E.2. If an individual Project Gutenberg™ electronic work is


derived from texts not protected by U.S. copyright law (does not
contain a notice indicating that it is posted with permission of the
copyright holder), the work can be copied and distributed to
anyone in the United States without paying any fees or charges.
If you are redistributing or providing access to a work with the
phrase “Project Gutenberg” associated with or appearing on the
work, you must comply either with the requirements of
paragraphs 1.E.1 through 1.E.7 or obtain permission for the use
of the work and the Project Gutenberg™ trademark as set forth
in paragraphs 1.E.8 or 1.E.9.

1.E.3. If an individual Project Gutenberg™ electronic work is


posted with the permission of the copyright holder, your use and
distribution must comply with both paragraphs 1.E.1 through
1.E.7 and any additional terms imposed by the copyright holder.
Additional terms will be linked to the Project Gutenberg™
License for all works posted with the permission of the copyright
holder found at the beginning of this work.

1.E.4. Do not unlink or detach or remove the full Project


Gutenberg™ License terms from this work, or any files
containing a part of this work or any other work associated with
Project Gutenberg™.
1.E.5. Do not copy, display, perform, distribute or redistribute
this electronic work, or any part of this electronic work, without
prominently displaying the sentence set forth in paragraph 1.E.1
with active links or immediate access to the full terms of the
Project Gutenberg™ License.

1.E.6. You may convert to and distribute this work in any binary,
compressed, marked up, nonproprietary or proprietary form,
including any word processing or hypertext form. However, if
you provide access to or distribute copies of a Project
Gutenberg™ work in a format other than “Plain Vanilla ASCII” or
other format used in the official version posted on the official
Project Gutenberg™ website (www.gutenberg.org), you must, at
no additional cost, fee or expense to the user, provide a copy, a
means of exporting a copy, or a means of obtaining a copy upon
request, of the work in its original “Plain Vanilla ASCII” or other
form. Any alternate format must include the full Project
Gutenberg™ License as specified in paragraph 1.E.1.

1.E.7. Do not charge a fee for access to, viewing, displaying,


performing, copying or distributing any Project Gutenberg™
works unless you comply with paragraph 1.E.8 or 1.E.9.

1.E.8. You may charge a reasonable fee for copies of or


providing access to or distributing Project Gutenberg™
electronic works provided that:

• You pay a royalty fee of 20% of the gross profits you derive from
the use of Project Gutenberg™ works calculated using the
method you already use to calculate your applicable taxes. The
fee is owed to the owner of the Project Gutenberg™ trademark,
but he has agreed to donate royalties under this paragraph to
the Project Gutenberg Literary Archive Foundation. Royalty
payments must be paid within 60 days following each date on
which you prepare (or are legally required to prepare) your
periodic tax returns. Royalty payments should be clearly marked
as such and sent to the Project Gutenberg Literary Archive
Foundation at the address specified in Section 4, “Information
about donations to the Project Gutenberg Literary Archive
Foundation.”

• You provide a full refund of any money paid by a user who


notifies you in writing (or by e-mail) within 30 days of receipt that
s/he does not agree to the terms of the full Project Gutenberg™
License. You must require such a user to return or destroy all
copies of the works possessed in a physical medium and
discontinue all use of and all access to other copies of Project
Gutenberg™ works.

• You provide, in accordance with paragraph 1.F.3, a full refund of


any money paid for a work or a replacement copy, if a defect in
the electronic work is discovered and reported to you within 90
days of receipt of the work.

• You comply with all other terms of this agreement for free
distribution of Project Gutenberg™ works.

1.E.9. If you wish to charge a fee or distribute a Project


Gutenberg™ electronic work or group of works on different
terms than are set forth in this agreement, you must obtain
permission in writing from the Project Gutenberg Literary
Archive Foundation, the manager of the Project Gutenberg™
trademark. Contact the Foundation as set forth in Section 3
below.

1.F.

1.F.1. Project Gutenberg volunteers and employees expend


considerable effort to identify, do copyright research on,
transcribe and proofread works not protected by U.S. copyright
law in creating the Project Gutenberg™ collection. Despite
these efforts, Project Gutenberg™ electronic works, and the
medium on which they may be stored, may contain “Defects,”
such as, but not limited to, incomplete, inaccurate or corrupt
data, transcription errors, a copyright or other intellectual
property infringement, a defective or damaged disk or other
medium, a computer virus, or computer codes that damage or
cannot be read by your equipment.

1.F.2. LIMITED WARRANTY, DISCLAIMER OF DAMAGES -


Except for the “Right of Replacement or Refund” described in
paragraph 1.F.3, the Project Gutenberg Literary Archive
Foundation, the owner of the Project Gutenberg™ trademark,
and any other party distributing a Project Gutenberg™ electronic
work under this agreement, disclaim all liability to you for
damages, costs and expenses, including legal fees. YOU
AGREE THAT YOU HAVE NO REMEDIES FOR NEGLIGENCE,
STRICT LIABILITY, BREACH OF WARRANTY OR BREACH
OF CONTRACT EXCEPT THOSE PROVIDED IN PARAGRAPH
1.F.3. YOU AGREE THAT THE FOUNDATION, THE
TRADEMARK OWNER, AND ANY DISTRIBUTOR UNDER
THIS AGREEMENT WILL NOT BE LIABLE TO YOU FOR
ACTUAL, DIRECT, INDIRECT, CONSEQUENTIAL, PUNITIVE
OR INCIDENTAL DAMAGES EVEN IF YOU GIVE NOTICE OF
THE POSSIBILITY OF SUCH DAMAGE.

1.F.3. LIMITED RIGHT OF REPLACEMENT OR REFUND - If


you discover a defect in this electronic work within 90 days of
receiving it, you can receive a refund of the money (if any) you
paid for it by sending a written explanation to the person you
received the work from. If you received the work on a physical
medium, you must return the medium with your written
explanation. The person or entity that provided you with the
defective work may elect to provide a replacement copy in lieu
of a refund. If you received the work electronically, the person or
entity providing it to you may choose to give you a second
opportunity to receive the work electronically in lieu of a refund.
If the second copy is also defective, you may demand a refund
in writing without further opportunities to fix the problem.

1.F.4. Except for the limited right of replacement or refund set


forth in paragraph 1.F.3, this work is provided to you ‘AS-IS’,
WITH NO OTHER WARRANTIES OF ANY KIND, EXPRESS
OR IMPLIED, INCLUDING BUT NOT LIMITED TO
WARRANTIES OF MERCHANTABILITY OR FITNESS FOR
ANY PURPOSE.

1.F.5. Some states do not allow disclaimers of certain implied


warranties or the exclusion or limitation of certain types of
damages. If any disclaimer or limitation set forth in this
agreement violates the law of the state applicable to this
agreement, the agreement shall be interpreted to make the
maximum disclaimer or limitation permitted by the applicable
state law. The invalidity or unenforceability of any provision of
this agreement shall not void the remaining provisions.

1.F.6. INDEMNITY - You agree to indemnify and hold the


Foundation, the trademark owner, any agent or employee of the
Foundation, anyone providing copies of Project Gutenberg™
electronic works in accordance with this agreement, and any
volunteers associated with the production, promotion and
distribution of Project Gutenberg™ electronic works, harmless
from all liability, costs and expenses, including legal fees, that
arise directly or indirectly from any of the following which you do
or cause to occur: (a) distribution of this or any Project
Gutenberg™ work, (b) alteration, modification, or additions or
deletions to any Project Gutenberg™ work, and (c) any Defect
you cause.

Section 2. Information about the Mission of


Project Gutenberg™
Project Gutenberg™ is synonymous with the free distribution of
electronic works in formats readable by the widest variety of
computers including obsolete, old, middle-aged and new
computers. It exists because of the efforts of hundreds of
volunteers and donations from people in all walks of life.

Volunteers and financial support to provide volunteers with the


assistance they need are critical to reaching Project
Gutenberg™’s goals and ensuring that the Project Gutenberg™
collection will remain freely available for generations to come. In
2001, the Project Gutenberg Literary Archive Foundation was
created to provide a secure and permanent future for Project
Gutenberg™ and future generations. To learn more about the
Project Gutenberg Literary Archive Foundation and how your
efforts and donations can help, see Sections 3 and 4 and the
Foundation information page at www.gutenberg.org.

Section 3. Information about the Project


Gutenberg Literary Archive Foundation
The Project Gutenberg Literary Archive Foundation is a non-
profit 501(c)(3) educational corporation organized under the
laws of the state of Mississippi and granted tax exempt status by
the Internal Revenue Service. The Foundation’s EIN or federal
tax identification number is 64-6221541. Contributions to the
Project Gutenberg Literary Archive Foundation are tax
deductible to the full extent permitted by U.S. federal laws and
your state’s laws.

The Foundation’s business office is located at 809 North 1500


West, Salt Lake City, UT 84116, (801) 596-1887. Email contact
links and up to date contact information can be found at the
Foundation’s website and official page at
www.gutenberg.org/contact

Section 4. Information about Donations to


the Project Gutenberg Literary Archive
Foundation
Project Gutenberg™ depends upon and cannot survive without
widespread public support and donations to carry out its mission
of increasing the number of public domain and licensed works
that can be freely distributed in machine-readable form
accessible by the widest array of equipment including outdated
equipment. Many small donations ($1 to $5,000) are particularly
important to maintaining tax exempt status with the IRS.

The Foundation is committed to complying with the laws


regulating charities and charitable donations in all 50 states of
the United States. Compliance requirements are not uniform
and it takes a considerable effort, much paperwork and many
fees to meet and keep up with these requirements. We do not
solicit donations in locations where we have not received written
confirmation of compliance. To SEND DONATIONS or
determine the status of compliance for any particular state visit
www.gutenberg.org/donate.

While we cannot and do not solicit contributions from states


where we have not met the solicitation requirements, we know
of no prohibition against accepting unsolicited donations from
donors in such states who approach us with offers to donate.

International donations are gratefully accepted, but we cannot


make any statements concerning tax treatment of donations
received from outside the United States. U.S. laws alone swamp
our small staff.

Please check the Project Gutenberg web pages for current


donation methods and addresses. Donations are accepted in a
number of other ways including checks, online payments and
credit card donations. To donate, please visit:
www.gutenberg.org/donate.

Section 5. General Information About Project


Gutenberg™ electronic works
Professor Michael S. Hart was the originator of the Project
Gutenberg™ concept of a library of electronic works that could
be freely shared with anyone. For forty years, he produced and
distributed Project Gutenberg™ eBooks with only a loose
network of volunteer support.

Project Gutenberg™ eBooks are often created from several


printed editions, all of which are confirmed as not protected by
copyright in the U.S. unless a copyright notice is included. Thus,
we do not necessarily keep eBooks in compliance with any
particular paper edition.

Most people start at our website which has the main PG search
facility: www.gutenberg.org.

This website includes information about Project Gutenberg™,


including how to make donations to the Project Gutenberg
Literary Archive Foundation, how to help produce our new
eBooks, and how to subscribe to our email newsletter to hear
about new eBooks.

You might also like